0% found this document useful (0 votes)
642 views

Quantitative Part 2 Solution - 1

1. The document provides solutions to math problems related to numbers, factors, primes, and divisibility. 2. It explains the logic and steps taken to solve problems involving operations on numbers such as finding factors, determining if a number is prime, finding the LCM or HCF of numbers, and testing for divisibility. 3. Many problems require determining unknown values in number expressions based on properties of numbers like being divisible by certain numbers or having a sum of digits that equals a particular value.

Uploaded by

Deeksha Kapoor
Copyright
© © All Rights Reserved
Available Formats
Download as PDF, TXT or read online on Scribd
0% found this document useful (0 votes)
642 views

Quantitative Part 2 Solution - 1

1. The document provides solutions to math problems related to numbers, factors, primes, and divisibility. 2. It explains the logic and steps taken to solve problems involving operations on numbers such as finding factors, determining if a number is prime, finding the LCM or HCF of numbers, and testing for divisibility. 3. Many problems require determining unknown values in number expressions based on properties of numbers like being divisible by certain numbers or having a sum of digits that equals a particular value.

Uploaded by

Deeksha Kapoor
Copyright
© © All Rights Reserved
Available Formats
Download as PDF, TXT or read online on Scribd
You are on page 1/ 45

Solutions for SM1002108

Chapter – 1 15. A number divisible by 11 must have the sum of its odd digits
(Numbers – ) and the sum of its even digits equal, or else their difference
should be a multiple of 11. Only choice (C) satisfies this
Concept Review Questions condition. Choice (C)

Solutions for questions 1 to 75: 16. Sum of the digits of 7654321A = 28 + A, so it must be
divisible by 9. As 0  A  9, 28  28 + A  37. Only when
1. 36 = 729 Ans : (729) 28 + A = 36 is the number divisible by 9.
10
 A = 8. Ans : (8)
2. 2 = 1024
215 = (210) (25) = (1024) (32) = 32768. Ans : (32768) 17. Sum of digits of 24687x = 27 + x
This is divisible by 9 when x = 0 or 9.
3. The sum of an even number of odd numbers is always even.  value of x cannot be determined uniquely. Choice (D)
Choice (A)
18. The number formed by the last 5 digits of PQRSTU6736
4. The product of 2 or more even numbers is always even. must be divisible by 32. When U = 1, this condition is
Choice (A) satisfied. When U = 2, this condition is not satisfied.
 We can’t say. Choice (C)
5. If all the numbers are even, the sum is even. If one of them
is even and the others are odd, the sum is odd. 19. The number can be written as 10000 (PQRST) + 9875
 We cannot say. Choice (C) As 10000 is divisible by 625
10000 (PQRST) is divisible by 625, while 9875 is not divisible
6. If all of them are odd, the product is odd. If one of them is by 625.
even, the product is even.  The number is not divisible by 625.
 we cannot say. Choice (C) Note: For a number to be divisible by 625, the number
formed by its last 4 digits must be divisible by 625.
7. If at least one of them is 2, the product is even, otherwise the Choice (B)
product is odd.
 we cannot say. Choice (C) 20. Take any number. Find the sum of its digits and subtract the
sum from the number. The result is always divisible by 9.
8. 35 is odd Choice (B)
If the sum of an odd number of prime numbers is even,
then one of them is always 2. Ans : (2) 21. Let the other number be x
(LCM) (HCF) = product of the numbers
9. 13013 = 13 (1001) = 13 (13) (11) (7) (264) (2) = (22) (x)  x = 24 Ans : (24)
= 132  (11)  (7)
22. If the LCM of two or more numbers equals their product, the
 13013 has 3 distinct prime factors. Ans : (3)
numbers must be co-prime, hence the HCF of any
two numbers would be 1. In the given problem,
10. Let x  0  277 LCM (x, y, z) = x.y.z
10x  2  77  (1)  HCF (y, z) = 1. Choice (A)

100x  27  77  (2) 23. HCF (2, 3, 5) = 1 and LCM (2, 3, 5) = 2 (3) (5)
Subtracting (1) from (2), we get 90x = 25 But HCF (2, 3, 6) = 1 and LCM (2, 3, 6)  2 (3) (6)
5  We can’t say. Choice (C)
 x Choice (A)
18 24. (38)  (64) = (38)  (2)4 x (3)4 = (312)  (24)
Number of factors of (38) (64) = (12 + 1) (4 + 1) = 65.
11. Let x = 0. 4 56 Ans : (65)

1000 x = 456  56 -------- (1) 25. (33) (77) (215) = (33) (77) (7  3)5 = (38)  (712)
10x = 4.56 -------- (2) The index of each prime factor is even.
Subtracting (2) from (1), we get (33) (77) (215) is a perfect square. Choice (A)
990x = 452
26. As the number has an odd number of factors, when the
226
x . Choice (D) number is expressed as a product of powers of prime factors,
495 each index is even. If each index is divisible by 6, then the
number is a perfect cube. Otherwise it is not a perfect cube.
12. Let x  0  123 We can’t say. Choice (C)

100x  12  3 -------- (1) 27. A perfect square has an odd number of factors
 the number is not a perfect square. Choice (B)
1000x  123  3 -------- (2)
Subtracting (1) from (2), we get 28. The square of a number consisting of n ones (1  n  9)
900x = 111 equals 1234….(n) (n  1) (n  2) …..1.
37 In the given problem, n = 5
x . Choice (C)  111112 = 123454321 Ans : (123454321)
300
29. (58) (710) is a perfect square
13. 231 is not prime. Number of ways of expressing it as a product of two distinct
 229 and 231 cannot be twin primes. Choice (D) (8  1)(10  1)  1
natural numbers=  49 Choice (C)
14. 437 = (19) (23) 2
323 = (19) (17)
30. (36) (73) can be written as a product of 2 distinct natural
567 = 7 (81)
651 = (21) (31) numbers in
6  13  1 or 14 w ays. Ans : (14)
241 is prime. Choice (D) 2
Triumphant Institute of Management Education Pvt. Ltd. (T.I.M.E.) HO: 95B, 2nd Floor, Siddamsetty Complex, Secunderabad – 500 003.
Tel : 040–40088400 Fax : 040–27847334 email : [email protected] website : www.time4education.com SM1002162/1
31. (26) (310) can be written as a product of 2 co-primes as
p3  r 3  q3  3pqr
(1)[(26)(310)] or (26) (310) i.e., in 2 ways.  pqr Choice (B)
Alternately, number of ways = 2 raised to the power of number p 2  q2  r 2  pq  qr  rp
of distinct prime factors minus 1 = 221 = 2. Choice (A)
46. If p + q + r = 0, p3 + q3 + r3 = 3pqr. Choice (A)
32. (23) (34) (56) (78) can be written as a product of 2 co-primes
in 24  1, i.e., 8 ways. Ans : (8) 47. 480 = 40  12
360 = 40  9
320 = 40  8
33. Sum of the factors of (24) (33) = 2  1  3  1  1240 .
5 4
LCM (480, 360, 320) = 40 LCM (12, 9, 8)
2 1 3 1
= (40) (72) = 2880. Ans : (2880)
Choice (B)
48. 63 = 21  3
34. Sum of the factors of 437 or (19) (23) 84 = 21  4
= 19  1  23  1  480 .
2 2
Ans : (480) 147 = 21  7
19  1 23  1 HCF (63, 84, 147) = 21. Ans : (21)

49. LCM 5 , 9 , 8   LCM5,9,8 =


35. A perfect number is half the sum of its factors. 360
 360.
Choice (B)  6 10 9  HCF 6,10,9 1
Ans : (360)
36. There are 213 odd numbers less than 214.
There are 213 numbers co-prime to it. Choice (D) 50. HCF  7 , 21, 14   HCF  7 , 21, 7   HCF 7,21,7

 12 5 18 
  
 12 5 9  LCM12,5,9
37. N = 3p + 2r 2q
7
Number of numbers less than N and co-prime to it = Choice (B)
180
= N1  1  1  1   N Choice (B)
 3  2 3 51. Yes, both expressions are equal to HCF (p, q, r, s)
Choice (A)
2
38. 289 = 17
Number of numbers co-prime to 289 and less than it = 288 – 52. Yes, both expressions are equal to LCM (p, q, r, s).
(Number of numbers having a common factor with 289) Choice (A)
= 288  16 = 272
(The only numbers less than 289 and not co-prime to 289 53. The given expression is in the form of a3  b3 , which
are 1  17, 2  17 ……… 16  17.) Ans : (272) a 2  b 2  ab
is always equal to a  b.
39. 48 = 243 Here, a = 1059 and b = 478.
Sum of the co-primes of 48 less than  The expression is a  b = 5.81. Ans : (5.81)
 1  1 1
 48 1   1    24  48 
48 = 384
( 48) 
2  2  3 3 54. Dividing 256 successively by 2, we get
Ans : (384) 2 256
2 128
2
 1 1 2 64
40.  x    x  2  2  4 (given)
2 2
 x  x 2 32
2 16
1
 x2   14 Choice (D) 2 8
x2
2 4
2 2 2
 1 1
41.  y    y2   2  3 2 (given) 1
 y  y2
  The number of twos in 256! is 1 + 2 + .4 + .... + 128
 y 2  1  11 Ans : (11) = 28  1 = 255. Ans : (255)
2
y 55. The remainder when a number is divided by 2n is equal to
the remainder when the ‘tail’ (the number formed by the last
42. x4  3x2 + 1 = (x2)2  2x2 + 1  x2 n digits of the given number) is divided by 2n. Here n = 3 and
= (x2  1)2  x2 = (x2  x  1) (x2 + x  1) the ‘tail’ is 677. Therefore, the remainder is 5. Therefore, the
 x2  x  1 and x2 + x  1 are both factors of x4  3x2 + 1. least number to be added such that the sum is divisible by 8
Choice (D) is 3. Ans : (3)

43. x3 + y3 = (x + y)3  3xy (x + y) 56. The product of any 6 consecutive natural numbers is always
= 123  3 (12) (18) = 1080. Ans : (1080) divisible by 6! i.e., 720. Choice (D)
57. The product of 10 consecutive even natural numbers is
44. If p3 + q3 + r3 = 3pqr, always divisible by 210  10! As each number is even, the
P3 + q3 + r3  3pqr = 0 product is divisible by 210. The other factors are
(p + q + r) (p2 + q2 + r2  pq  qr  rp) = 0 10 consecutive numbers. Their product is divisible by10!
1  The product is always divisible by 210  10!
(p + q + r) = 0 or [(p  q)2 + (q  r)2 + (r  p)2] = 0
2 Choice (C)
i.e., p + q + r = 0 or p = q = r. Choice (C) 58. The index of each prime factor must be even. If we multiply
the number by (5) (7) i.e., 35, the resulting indices are all
45. p + q + r  3pqr
3 3 3
even. Ans : (35)
= (p + q + r) (p2 + q2 + r2  pq  qr  rp)
Replacing q by – q on both sides of the equation above, 59. If we divide the number by (5) (7), the quotient is a perfect
p3  q3 + r3 + 3pqr square. This is the least number by which we have to divide.
Ans : (35)
= (p  q + r) (p2 + q2 + r2 + pq + qr  rp)
Triumphant Institute of Management Education Pvt. Ltd. (T.I.M.E.) HO: 95B, 2nd Floor, Siddamsetty Complex, Secunderabad – 500 003.
Tel : 040–40088400 Fax : 040–27847334 email : [email protected] website : www.time4education.com SM1002162/2
60. The index of each prime factor must become divisible by  The number has at least 27 digits and less than 30 digits.
3 upon division. The least number which achieves this Choice (D)
objective is (2) (32) i.e., 18. Choice (D)
73. Suppose a number x has m digits
61. Least perfect cube greater than 395 is 512. 117 should be i.e., 10m1  x < 10m
added to 395 to obtain 512. Ans : (117)  102m2  x2 < 102m
i.e., x2 has 2m or 2m  1 digits. Conversely, if a number has
62. 484 is the greatest perfect square below 500. 2m – 1 or 2m digits, its square root has m digits.
 16 is the least natural number to be subtracted from 500. Therefore, if a number has 13 digits, its square root has 7
Ans : (16) digits. Ans : (7)

63. Let the number be N. Let the quotient obtained, when the 74. (2PQR)4 must be at least (2000)4 and less than (3000)4
number is divided by 54, be q. (2000)4 as well as (3000)4 have 14 digits.
N = 54q + 31  (2PQR)4 has 14 digits. Choice (B)
When N is divided by 27, the quotient is 2q + 1 and the
remainder is 4. Ans : (4) 75. Suppose a number x has m digits
10m 1  x < 10m
64. All such numbers are of the form kLCM(7, 8) + 3. The least  103m  3  x3 < 103m
natural number of this kind occurs when k = 0. This number
i.e., x3 has 3m  2, 3m  1 or 3m digits, so if a number has
is 3. Choice (A)
28, 29 or 30 digits, its cube root has 10 digits.
Choice (A)
65. Let the least natural number be N. Let the number divided by
24 and 18 result in quotients of q1 and q2 respectively.
Exercise – 1(a)
N = 24q1 + 18 = 18q2 + 12 we get
N + 6 = 24 (q1 + 1) = 18 (q2 + 1) Solutions for questions 1 to 40:
 N + 6 is the least number divisible by 24 and 18 i.e., by
LCM (24, 18) = 72.  N = 66. Ans : (66) 1. Let the number be N.
Let N = DK + 13, where K is the quotient
66. Numbers which leave a remainder of 3 when divided by 3N = 3DK + 39
5 are 3, 8, 13, 18, 23, 28, . . . As the remainder of 3N divided by D is 2, 37 must go into the
Numbers which leave a remainder of 5 when divided by
37
6 are 5, 11, 17, 23, 29, . . . quotient in the form .
Therefore numbers of the form kLCM(5, 6) + 23 satisfy both D
the conditions. Putting k = 0, gives the least natural number. 3N = D  3k  37  + 2
Ans : (23)  
 D 
37
67. The length (in cm) of the side of the smallest square must be As must be a natural number, D can be 37.
divisible by 7 as well as 5. D
 It must be L.C.M (7, 5) or 35 cm. But as D exceeds the remainder when N is divided by D, D
Its area = 352 cm2 = 1225 cm2. Choice (A) can be 37 only. Ans : (1)

68. Required number = HCF (107  17, 78  18) = 30. 2. Any natural number having an even number of factors is not
Ans : (30) a perfect square. Any natural number that is not a perfect
square can be written as a product of two factors where one
69. Let the remainder in each case be r. of the factors lies between 1 and its -square root and the
Let the largest number be N and the quotients when N other factor lies between its square root and itself.
divides 34, 58 and 94 be q1, q2 and q3 respectively  In the given problem, both (a) and (b) are true.
34 = Nq1 + r  (1) Choice (A)
58 = Nq2 + r  (2)
94 = Nq3 + r  (3) 3. The remainder of X divided by 16 is equal to the remainder
when the number formed by the by the last 4 digit of X is
Subtracting (1) from (2), we get 24 = N (q2  q1)  (4)
divided by 16. We tabulate below the numbers, the number
Subtracting (1) from (3), we get 60 = N (q3  q1)  (5)
of numbers, the number of digits and the total number of
From (4) and (5), N divides 24 and 60.
digits in X
 N = HCF (24, 60) = 12. Choice (C)
Total
Number of Number of
70. The successive division is shown below Numbers number of
Numbers Digits
Digits
Number/ Quotient N q1 q2 19 9 9 19
5 6 7 10  54 45 90 99
Divisor 55 Parts of 1 number 1 100
+  + 
We see that the number formed by the last 4 digits of X is
Remainder 3 4 5 3545 (The 3 from 53, then 54 and the first 5 from 55)
Rem (3545/16) = 9 Ans : (9)
The least number = ((5  6) + 4) 5 + 3 = 173 4. Let P be 100a + 10b + c
Ans : (173) Q = 100c + 10b + a
71. Number of three digit natural numbers divisible by 8, Q  P = 99 (c  a)
12 and 15 = Number of three digit natural numbers divisible As Q  P is divisible by 5.
by LCM (8, 12, 15), i.e., 120. c  a is divisible by 5.
There are 8-three digit natural numbers divisible by 120, As Q > P, c  a is positive
120(1), 120(2), . . . 120(8). Ans : (8) c  a = 5, Hence (a, c) can be (1, 6), (2, 7), (3, 8) or (4, 9) or
P can be 1b6,2b7,3b8 or 4b9 where b can be any digit.
72. The number of digits in the product must be at least the  106  P  499
number of digits in (106) (109) (1011) and less than the Only choice (B) accommodates all the values of P.
number of digits in (107) (1010) (1012). Choice (B)
Triumphant Institute of Management Education Pvt. Ltd. (T.I.M.E.) HO: 95B, 2nd Floor, Siddamsetty Complex, Secunderabad – 500 003.
Tel : 040–40088400 Fax : 040–27847334 email : [email protected] website : www.time4education.com SM1002162/3
5. (i) 183 = (2  32)3 = 36 23. The number of factors is 7(4) = 28 N = KLCM(3, 5, 7, 9) –1.
The product of all these factors is (183)14 = 1842. As N is the least number, N = LCM (3, 5, 7, 9) –1
Choice (B) = 314. Ans : (314)
(ii) 12! = (2.4.6.8.10.12) (3.5.7.9.11)
= (210 . 32 . 5) (3.5 7.9.11) 13. Let the number be 8k + 3, where k = 0, 1, 2, 3, 4, ……
= 210 35 52 7 (11) When (8k + 3) is divided by 7, the remainder is 1.
The number of factors of 12! is 11(6) (3) (2) (2) = 792  (8k + 3  1), i.e. (8k + 2) is divisible by 7.
The smallest value of k for which 8k + 2 is divisible by 7 is 5.
792
The product of all these factors is 12! 2 .
 The smallest such number is 8  5 + 3 = 43.
The general form of the required number is 56p + 43
= (12!)396. Choice (B) (56 is the LCM of 8 and 7).
The largest five digit multiple of 56 is 99960.
6. If a number N = ap.bq . cr …… where a, b, c are prime  The required number is 99960 + 43  56 = 99947.
numbers and p, q, r …… are integers, then, the number of Choice (D)
different ways, in which N can be written as product of two
co-primes factors, is 2n-1 where ‘n’ is the number of different 14. The required number would divide 565  5, 847  7 and 1551
prime numbers used in resolving N into prime factors.  11 i.e. 560, 840 and 1540.
Here, N = 11025 = 25  441 Therefore, we have to find the HCF of 560, 840 and 1540.
= 52  212 = 32  52  72 560 = 24(5)(7)
Three different prime numbers i.e., 3, 5 and 7 are used in the 840 = 23(3)(5)(7)
resolution into prime factors. Hence, n = 3 1540 = 22(5)(7)(11)
2n-1 = 23-1 = 4  HCF of the three numbers = 22 (5)(7) = 140
i.e., 11025 can be written as product of a pair of co-prime Ans : (140)
factors in 4 different ways. Ans : (4)

7. Up to 1400, there are 200 multiples of 7. There are 280 15. Required time = LCM  3  7 5 , 3  1 1 , 3  5 2 
 
multiples of 5. There are 40 multiples of both 5 and 7 (i.e. of  6 3 3
LCM (5, 7) i.e. 35). = LCM  65 , 13 , 26 
 Up to 1400, there are 100 odd numbers divisible by 7,  6 3 3 
140 odd numbers divisible by 5, and 20 odd number 130 1
divisible by both 5 and 7.   43 seconds. Choice (D)
There are 140 + 100 – 20 i.e. 220 odd numbers which are 3 3
divisible by either 5 or 7.
 The remaining 700 – 220 i.e. 480 odd numbers are 16. Let the numbers be 11x and 11y, where x and y are relative
divisible by neither 5 nor 7. Ans : (480) primes and x  y.
LCM of 11x and 11y is 11 xy.
8. 840 = 23  3  5  7 11xy = 1001  xy = 91
Number of co-primes to 840 and less than it  x = 1 and y = 91
or x = 7 and y = 13
= 840 1  1  1  1  1  1  1  1   The numbers could be 11, 1001 or 77, 143.
 2  3  5  7 
Since the sum of the two numbers is 220, the required
= 840  1   2   4   6  = 192 number is 77. Ans : (77)
2 3 5 7
    
Sum of the co-primes to 840 and less than it 17. The procedure is as follows
(192) (840) 248) 480 (1
= = 80640 Choice (B)
2 248
---------
9. LCM of 15, 20, 25 and 30 is 300. 232) 248 (1
15 = 3(5) 232
20 = 22(5) --------
25 = 52 16) 232 (14
30 = 2(3)(5) 224
 LCM = 22(3)(52) = 300 --------
The buses would start together again after 300 minutes i.e. 8) 16 (2
16
after 5 hours from 8:00 a.m. i.e., 1:00 p.m. Choice (C)
-----
10. LCM of 10, 16 and 20 is 80. The number of sweets should 0
be of the form 80k + 1, where k = 1, 2, ….. It must also be  The numbers are 248 and 480. Choice (B)
divisible by 23. Also it is less than 200
80k + 1 < 200 18. Given function is n(n² + 20), n being an even number.
199 39 Let n = 2k, when k is any positive integer
k< i.e., 2 Hence, n(n² + 20) = 2k(4k² + 20)
80 80 = 8k (k² + 5) = 8k [(k² – 1) + 6]
k = 1 or 2. = 8k (k² – 1) + 48k
Only when k = 2, 80k + 1 is divisible by 23. The number of = 8 (k – 1) k (k + 1) + 48 k
sweets with me is 161. Ans : (161) = 8[multiple of 6] + 48 k
11. LCM of 7, 11 and 21 is 231. = 4.8 L + 48k, where L is an integer.
The number would be in the form 231k + 5. = 48 x an integer.
10164 is the smallest five digit multiple of 231. i.e., x (x² + 20) is always a multiple of 48, as long as n is even.
Hence, 48 is the HCF of all numbers represented by
The required number is 10164 + 5 = 10169.
n (n² + 20), n being even. Choice (D)
Choice (B)
12. Let the number be N. 19. Let the number be N. Let the quotient when the number is
N = 3k1 + 2 = 5k2 + 4 = 7k3 + 6 = 9k4 + 8 divided by 6 be q1,
N + 1 = 3(k1 + 1) = 5(k2 + 1) = 7(k3 + 1) = 9(k4 + 1) N = 6q1 + 4.
N + 1 = KLCM(3, 5, 7, 9) Let the quotient be q2 when q1 is divided by 7.

Triumphant Institute of Management Education Pvt. Ltd. (T.I.M.E.) HO: 95B, 2nd Floor, Siddamsetty Complex, Secunderabad – 500 003.
Tel : 040–40088400 Fax : 040–27847334 email : [email protected] website : www.time4education.com SM1002162/4
q1 = 7q2 + 5 Sum of all the factors of 33333333 =
N = 6 (7q2 + 5) + 4  32  1  112  1   1012  1   137 2  1 
= 42q2 + 34.        
 3 1   11  1   101  1   137  1 
When N is divided by 21, the quotient is 2q2 + 1 and the        
remainder is 13. Ans : (13)
 73 2  1 
 
20. The smallest number is {(4)(4) + 3}3 + 2 = 59  73  1 
 
The general form of the number would be 60k + 59.
60 = 3(4)(5) (3 + 1) (11 + 1) (101 + 1) (137 + 1) (73 + 1)
The greatest 4 digit multiple of 60 is 9960. = 4(12) (102) (138) (74)
Since 9960 + 59 = 10019 becomes a five-digit number, = 49997952. Choice (D)
10019  60 = 9959 is the required number. 26. The given function n (n2 – 4) (n4 – 10n2 + 9) can be written as:
Choice (D) n (n + 2) (n – 2) (n2 – 9) (n2 – 1)
21. 20 = 4(5) = n (n + 2) (n – 2) (n + 3) (n – 3) (n + 1) (n – 1)
The index of the greatest power (IGP) of 5, that can divide 200! = (n – 3) (n – 2) (n – 1) n (n + 1) (n + 2) (n + 3) ;
is 40 + 8 + 1 = 49 i.e. the function is the product of 7 consecutive positive
The IGP of 2 that can divide 200! is 100 + 50 + 25 + 12 + 6 integers. (As n > 3, n – 3 > 0); and hence, the function is
+ 3 + 1 = 197 divisible by 7!
 The IGP of 4 that can divide 200! is 98. 7! = 1(2)(3)(4)(5)(6)(7)
 The IGP of 20 that can divide 200! is 49. (The lower of the The given options are
two values) Ans : (49) 126 = 3(6)(7)
72 = 3(4)(6)
22. The number of zeros at the end of 175! is same as the 52 = 4(13)
greatest power of 5 in 175!. 175! has 35 fives, 7 twenty-fives and 144 = 2(3)(4)(6)
and 1 one twenty five. Except 52, all others are factors of 7!
 Index of the greatest power of 5 in 175! is (35 + 7 + 1) Hence 52 is the only number that is not a factor.
= 43. Ans : (43) Choice (C)

23. N = a2  b2 = (a + b) (a  b) [a > b] 27. A = 3(2n1 – 1) and B = 5(2n2 – 1)


For a and b to be natural numbers, a + b and a  b must be 5 A  3B (30n1  15 )  (30 n2  15 )
= = 2(n1 – n2)
of the same parity. 15 15
 We need to identify N for which the number of ways of 5 A  3B
expressing N as a product of 2 factors of the same parity is = n1 – n2
30
the least.
5A – B = 30n1 – 15 – 10n2 + 5 = 10(3n1 – n2 – 1)
Choice (A) 5A  B 5A  B 3n1  n 2  1
187 = 1  187 = 11  17  = 3n1 – n2 – 1 while =
10 20 2
 k(187) = 2 which may not be an integer. Choice (D)
Choice (B)
28. 72000 = 8(9)(23 53) = 26(32)(53)
120 = 2  60 = 4  30 = 6  20 = 10  12 We should multiply with 24 (33)(52)
 k(120) = 4 i.e. 10800 so that the product is a perfect 5th power.
Choice (C) Choice (A)
k(110) = 0 29. 36 + 37 + 38 + 39 + 40 = 190
Choice (D) 50(51)
1085 + 190 = 1275 =
105 = 3  35 = 5  21 = 7  15 2
 k(105) = 3 So the least of the numbers on the intact houses can be 36.
k(110) which is equal to 0, is the least. Choice (C) 26 + 27 + 28 + 29 + 30 = 140
49(50)
24. 1125 = (5) (225) = 5332 1085 + 140 = 1225 =
2
Number of factors of 1125 = (3 + 1) (2 + 1) = 12
1800 = 8(225) = 23(32) (52). So it can be 26 also.
Number of factors of 1800 = (3 + 1) (2 + 1) (2 + 1) = 36 Alternate method:
Number of common factors of 1125 and 1800 = Number of N(N  1)
factors of HCF (1125, 1800) i.e. of 225 (i.e. 3252) The sum of the numbers on all the houses is .
= (2 + 1) (2 + 1) = 9. 2
Number of factors of 1125 which are not factors of 1800 Let the numbers on the houses that remained intact be
= 12 – 9 = 3. M – 2, M – 1, M, M + 1, M + 2
Number of factors of 1800 which are not factors of 1125 The sum of these 5 numbers is 5M.
= 36 – 9 = 27.  The sum of the numbers on the destroyed houses is
Number of factors of only one of 1125 and 1800 = 3 + 27 N(N  1)
– 5M.
= 30. Ans : (30) 2
N(N  1)
25. Let X = 33333333 = 3(11111111) = 3(11110000 + 1111)  – 5M = 1085
= 3(1111)(10000 + 1) 2
= 3(1111)(10001).  N(N+1) = 10M + 2170 = 10(M – 2) + 2190.
We can recognize 10,001 as the difference of two squares The values of M – 2 (the least of the numbers on the intact
i.e. 10, 001 = 11025 – 1024 = 1052 – 322 = 137 (73). houses suggested in the options and the corresponding
 X = 3(11) (101) (137) (73) values of N(N + 1) and N (for those values of M – 2 which
Sum of all the factors of p1a p b2 p c3 ………where p1, p2, produce an integral value of N) are tabulated below.
p3…..are primes and a, b, c…….are whole numbers is M–2 N(N+ 1) N
 p1a  1  1   p b2  1  1   p c3  1  1  25 2440 -
      …. 26 2450 49
 p1  1   p 2  1   p 3  1 
      36 2550 50
 M – 2 can be 26 or 36. Choice (D)
Triumphant Institute of Management Education Pvt. Ltd. (T.I.M.E.) HO: 95B, 2nd Floor, Siddamsetty Complex, Secunderabad – 500 003.
Tel : 040–40088400 Fax : 040–27847334 email : [email protected] website : www.time4education.com SM1002162/5
30. The factors of N, which are perfect cubes will be of the form 35. The successive expressions have been relabeled as shown
= 2a (3b)(5c), where a can be 0, 3, 9.,…… 24, b can be 0, 3, below.
9, … 15, and c can be 0, 3, … 15. The number of factors F(10, F(9, F(8, F(7, F(6, i)))) = 1
which are perfect cubes is A B C D
= 9(6)(6) = 324. Choice (A)  F(10, A) = 1  0 < A  10
 0 < F(9, B)  10  0 < B  10(9)
 7 7 7   0 < F(8, C)  10(9)  0 < C  10(9)(8)
31. X =  , , ,..... 7(512 ) .
 0 < F(7, D)  10(9)(8)  0 < D  10(9)(8)(7)
 128 64 32 
 0 < F(6, i)  10(9)(8)(7)  0 < i  10(9)(8)(7)(6)
 
Let A = 2 7 , 2 6 , 2 5 ,........ 29 and  i can have all values greater than 0 and up to 30, 240.
 
let B = 7,  6,.......... 9
(i) is false.
(ii) is false. Choice (D)
The 7+1+9 (viz 17) elements of B can be arranged as shown.
36. P1 = {1, 2, 3, 4, 5, 6}
–7 –6 –5 –4 –3 –2 –1 0 1 P2 = {2, 3, 4, 5, 6, 7}
9 8 7 6 5 4 3 2 P3 = {3, 4, 5, 6, 7, 8}
From each of the 9 columns, we can select only one element. :
If we select both, the numbers in the (say) first column, the :
corresponding numbers in A would be 2–7 and 29 and the
corresponding elements in X would be 7(2–7), 7(29). They P8 = {8, 9, 10, 11, 12, 13}
would have a product of 4(49), i.e. 196. The sets P1 and P2 do not contain 8 or its higher multiple.
The sets P3 to P8 contain a multiple of 8.
 The subset Y can have at the most 9 elements.
Ans : (9)  In the first 8 sets, 6 contain a multiple of 8.
Similarly, it can be shown that for each collection of
32. If the sum of 3 numbers x, y and z is constant (in this case 8 successive sets, 6 sets contain a multiple of 8 while the
9m + 10), x2 + y2 + z2 (say s) will have its minimum value other 2 don’t.
when x, y and z are as close to each other as possible, i.e. x  The total number of sets which contain a multiple of
= 3m + 3, y = 3m + 3 and z = 3m + 4 Therefore, the minimum 8 =
88
6 66 . Ans : (66)
value of S is 8
(3m + 3)2 + (3m + 3)2 + (3m + 4)2 or 27m2 + 60m + 34.
Choice (C)
x x  x  3 27 x  3
 x  3
37. y =
33. The following results are useful in all such problems. 2

If the index of the greatest power, (IGP), of p in A is m and


x  27  x   3
3
the IGP of p in B is n, then x 3
(1) the IGP of p in AB is m + n. =  = x 3 x 9
(2) if m  n, the IGP of p in A + B is the smaller of m and x 3 x 3
n.
If m = n, the IGP of p in A + B could be M or more than m.
= x – 2  3  x + 2.25 + 6.75 =
 
 x  1.5  6.75
2

2
Let S = 64! + 65! + 66! + 67! + …..+120! It can be seen that y is an increasing function for all non
The IGPs (Index of Greatest power) of 2 in successive negative values of x
terms are 63, 63, 64, 64, 66, 66, …..
 when x = 25 and x = 49, the minimum and the maximum
We have to express S as S1 + S2.
values of y for x in the given range are obtained as 19 and
Here, we have to club the first four forms.
37 respectively.
S1 = 64! [1 + 65 + 65(66) + 65(66)(67)], the expression in
 y satisfies 19  y  38 Choice (B)
the bracket is a multiple of 2 but not of 4.
The IGP of 2 in 64! is 63.
38. We should take the smallest 5 digit number and the
The IGP of 2 in the bracket is 1.
greatest 4 digit number for the difference to be the least.
 The IGP of 2 in S1 is 64 which the IGP of 1 in S2 is 66.
The required difference = 12345  9876 = 2469
 The IGP of 2 in S is 64. Ans : (64)
Ans : (2469)
34. p = q + 2 = r + 4.  p = q + 2, q = r + 2.
Each prime number greater than 3 is of the form 6k + 1. 39. We can write 540 as 5 (2) (6) (9) or (5) (4) (3) (9) since the
If r is of the form 6k + 1, than q is of the form 6k + 3 and digits are all distinct
then q is divisible by 3. Also q is prime.  Only possible So number of four digit numbers
value of q is 3. But then r(= 1 and hence) will not be prime. = 4! + 4! = 48. Ans : (48)
If r is of the form 6k –1, p is of the form 6k + 3. Only 40. If p is any prime number, (p – 1)! is not divisible by p.
possible value of p is 3. But then q(= 1 and hence) will not 1  P  40
be prime. Each prime value of P satisfies the given condition.
r is neither of the form 6k + 1 or 6k – 1. r is not a prime There are 12 such values.
number greater than 3. When P is 1, (P – 1)! = 1 (which is divisible by P).
r = 2 or 3. The only composite number satisfying the condition is 4.
If r = 2, then q = 4 which is not prime. There are 13 values of P satisfying the condition.
 r = 3 and q = 5 and p = 7. Choice (D)
Only one combination exists for p, q, r.
Exercise – 1(b)
Alternative Solution:
r, q, p is an increasing AP with common difference 2. Solutions for questions 1 to 60:
For any 3 terms in an AP, with common difference which is
not a multiple of 3, one of the numbers is a multiple of 3, 1. p is a prime number greater than 3, hence it can be
another leaves a remainder of 1 and the third leaves a represented by either (6k + 1) or (6k – 1), k being a positive,
remainder of 2 (when divided by 3). The only way in which integer.
all 3 can be prime is when the multiple of 3 is 3 itself. Hence p2 – 1 = [(6k + 1)2 – 1] or [(6k – 1)2 – 1]
i.e. (r, q, p) = (3, 5, 7). We can also consider (– 7, –5, –3). Consider (6k + 1)2 – 1:
As primes are considered to be positive, we have only one This is equal to [(6k + 1) + 1] [(6k + 1) – 1]
combination. Ans : (1) = (6k + 2) (6k) = 12k (3k + 1)  (1)

Triumphant Institute of Management Education Pvt. Ltd. (T.I.M.E.) HO: 95B, 2nd Floor, Siddamsetty Complex, Secunderabad – 500 003.
Tel : 040–40088400 Fax : 040–27847334 email : [email protected] website : www.time4education.com SM1002162/6
when ‘k’ is odd, 3k + 1 is even, hence k (3k + 1) is even; and (a  1) (a  2) . . . . (a  b)
= = an integer
when k is even. k (3k + 1) is even; (1) (2) . . . (b)
 k (3k + 1) is always divisible by 24  (2)
The product of b consecutive natural numbers is always
when p = 6k – 1
divisible by b! Choice (D)
p2 – 1 = 12k (3k – 1) and this is also divisible by 24, for all
values of k.
10. 1  k  40
Hence, for all prime numbers which are greater than 3,
 5  5k  200
(p2 – 1) is always divisible by 24.
Hence, 6 and 2 are also factors of (p2 – 1). Choice (D)  6  5k + 1  201
 All the prime numbers between 6 and 201, which are of
2. When 478185 is divided by 19, the remainder is 12. form 5k + 1 are 11, 31, 41, 61, 71, 101, 131, 151, 181 and
Ans : (12) 191 i.e., 10 in all. Ans : (10)

3. A zero at the end of a product comes from the product of 11. Required divisor = 238 + 342  156 = 424. Choice (C)
2 and 5. None of the prime numbers except 2 is even. None
of the prime numbers except 5 is divisible by 5. 12. As each digit of S is even, the first two digits must be
 The product ends with 1 zero. Ans : (1) 2 each. Their sum is 4. The other two digits, being even,
must have an even sum. As the sum of the digits of S is
4. Let the least number to be added to 1648 so that a remainder divisible by 3, the sum of the last two digits can be 2 or 8 or
of 10 is left when the resulting number is divided by 14 or 21 14. (The last two digits are even, each at most must be 8.
be ‘x’.  Their sum at most can be 16).
1648 + x = k(L.C.M (14, 21)) + 10. If the sum of the last two digits is 2, the third digit must be
= 42k + 10. 2 and the fourth digit must be 0 or vice versa.
 1638 + x = 42k. ----------- (1) x has 2 possibilities. If the sum of the last two digits is 8, (third
As ‘x’ is a natural number, x>0. digit, fourth digit can be (0, 8), (2, 6), (4, 4), (6, 2) or (8, 0)
42k > 1638.  x has five possibilities.
If the sum of the last two digits is 14, the third digit can be 6
1638
k> and the fourth digit can be 8 or vice versa.
42  x has 2 possibilities.
 k > 39.  x has a total of 9 possibilities. Ans : (9)
So, the least value of k is 40.
Substituting k = 40 in (1), 13. (i) 900 = 22  32  52
x = 42 (40) – 1638 = 1680 – 1638 = 42 Number of factors (divisors) = (2 + 1) (2 + 1) (2 + 1)
x = 42. = 3  3  3 = 27.
 Required number is 42. Choice (B) Number of divisors excluding ‘1’ and itself = 27 – 2 = 25
Choice (A)
5. N3 – N = N (N2 – 1) = N (N – 1) (N + 1)
= (N – 1) N (N + 1) = Product of 3 consecutive integers; which (ii) Number of ways in which 1500 can be expressed as a
is divisible by 6.i.e. (N3 – N) is divisible by 6, when N > 1. product of two of its factors
Therefore, the remainder is zero. Number of factors of 1500
Hence, the product of the two remainders is zero. =
2
Ans : (0) 1500 = 3  53  22
Number of factors of 1500 = (1 + 1) (3 + 1) (2 + 1) = 24.
6. Let x = 10. 04343. . . . Number of ways in which 1500 can be expressed as a
 10x = 100.4343 --------- (1)
24
1000x = 10043.4343 --------- (2) product of two of its factors = = 12.
(2)  (1)  990x = 9943 2
9943 Choice (B)
x= Choice (D)
990 14. Given number = 784
When resolved into primes factors,
7. Let the number be 784 = 4  196 = 22  142 = 24  72
1000x + 100y + 10z + u The number of ways in which N can be expressed as product
Let another four digit number formed by permuting its digits of a pair of different factors is
be 1000y + 100z + 10u + x
The difference between these two is 999x  900y  90z  9u
1
p  1q  1.......  1  where N = ap.bq . cr ……, a, b, c
2
i.e. 9(111x  100y  10z  u) which is always a multiple of 9.
Let us consider another four digit number with the same being prime factors of N and p, q, r are whole numbers.
digits, 1000z + 100u + 10x + y. Hence, the answer to the question is
The difference is 990x + 99y  900z  99u 1
4  12  1  1   1 (14 )  7 Ans : (7)
i.e. 9(110x + 11y  110z  11u) which again would be a 2 2
multiple of 9. Choice (D)
15. 4! 5! = 24 (120) = 23(3) 23(3) (5) = 26 32 51
8. n7 – n = n(n6 – 1) = n(n3 – 1) (n3 + 1) The number of factors is 7(3)(2) = 42
= n(n – 1) (n2 + n + 1) (n + 1) (n2 – n +1) The product of all these factors is [24(120)]21 = (2880)21
When n = 1, n7–n = 0 is divisible by all numbers. Choice (C)
When n = 2, n2 + n + 1 = 7
 n7 – n is divisible by 7 16. 8998 = 227316. The number of factors is 28(17) = 476
When n = 3, n2 – n + 1 = 7  n7 – n is divisible by 7. The product of all these factors is (227316)238 = 26426  33808.
Similarly for n = 4, 5, 6, and 7, n7 – n is divisible by 7. Choice (D)
From n = 8 onwards the same pattern repeats.
n7 – n is always divisible by 7. Choice (C) 17. 8640 = 26  33  5
Number of co-primes to it and less than it
= 8640 1  1  1 1  1  1  = 8640  1   2   4  = 2304
9. As a < (a + b) and b < (a + b), (a + b)! is divisible by both a!
and b!. 2 3 5
 2  3  5    
(a  b )! (1) ( 2) . . . (a) (a  1) . . . (a  b)
= Ans : (2304)
a! b! (1) ( 2) . . . (a) (1) ( 2) . . . (b)
Triumphant Institute of Management Education Pvt. Ltd. (T.I.M.E.) HO: 95B, 2nd Floor, Siddamsetty Complex, Secunderabad – 500 003.
Tel : 040–40088400 Fax : 040–27847334 email : [email protected] website : www.time4education.com SM1002162/7
18. 3 m 78 cm = 378 cm  21 31 41  1
4 m 80 cm = 480 cm = HCF  lb, lb, lb   lb
378 = 2  33  7  4 4 5  20
480 = 25  3  5 Number of guests = Number of pieces
HCF of 378 and 480 = 2  3 = 6 21 31 41
 Each side of the square tile = 6 cm.
378  480
4  4  5  424
 Minimum number of tiles required = = 5040 = Ans : (424)
1 1 1
66
20 20 20
Choice (B)
24. Complete remainder is the smallest number which when
900!
19. = (900) (899) (898)…..(451). successively divided by 7, 11 and 5, the respective
450! remainders are 5, 1 and 1.
The index of the greatest power (IGP) of 11 that divides i.e. {(1  11) + 1}  7 + 5 = 89 Ans: (89)
900!
is obtained as below.
450! 25. Let the quotients obtained when the number is successively
900 81 7 divided by 4, 5 and 6 be denoted by K1, K2 and K3 respectively.
Let the number be denoted by N.
11 11
N = 4K1 + 3
450 40 3 K1 = 5K2 + 4
11 11 K2 = 6K3 + 5
N = 4(5K2 + 4) + 3
IGP of 11 in 900! is 81 + 7 i.e. 88 and the IGP of 11 in 450! = 20K2 + 19
is 40 + 3 i.e. 43. = 20(6K3 + 5) + 19
900! = 120K3 + 119
IGP of 11 in = 88 – 43 = 45. Ans : (45)
450! When K3 = 0, the smallest value of N is obtained as 119.
When K3 = 1, the second smallest value of N is obtained as 239.
20. Divisors are : 5, 7 and 11.
Remainders are : 4, 6, and 10. (i) Hundreds digit of 119 is 1. Choice (A)
Complements are : 1, 1, and 1. (ii) Hundreds digit of 239 is 2. Choice (A)
Hence, (LCM of 5, 7 and 11)K–1 is the general form of the
selection. 26. A number whose units digit equals the units digit of its square
 (5 x 7 x 11 x k) – 1 or (385 K – 1) is the number that must end with 0, 1, 5 or 6.
satisfies the above condition, K being a position integer. In the given problem, AB ends with a digit which is the same
The required number shell be a multiple of 17; let it be 17Q, as the units digit of its square.
when Q is a positive integer.  B must be 0, 1, 5 or 6.
Hence, 385 K – 1 = 17Q. The square of any number ending with 0 must end with
385K  1 2 zeroes. If B = 0, (AB)2 cannot be CCB (it can be CBB)
 Remainder of is zero
17 B0
11K  1  B is 1 or 5 or 6.
 Remainder of 22K + is zero As CCB is a three-digit number, its square root i.e. AB must
17
be from 10 to 31.
11K  1
 Remainder of is zero If B = 1, AB = 11 or 21 or 31.
17 (AB)2 will be of the form CCB only when AB = 21.
 11K – 1 is a multiple of 17. If B = 5, AB = 15 or 25
By trial and error, it can be seen that when (AB)2 will be of the form CCB only when AB = 15
K is 14, 11K – 1 = 154 – 1 If B = 6, AB = 16 or 26
= 153 which is a multiple of 17. (AB)2 can never be of the form CCB.
And K = 14 is the last value of K that satisfies the above  The sum of all the possible values of CCB
equation. = 212 + 152 = 666 Ans : (666)
Hence, the least value of the number is
385K – 1 = 17 22K  11K  1  27. a3 + b3 = (a + b)3  3ab(a + b)
  = (11)3  3  7  11
 17 
= 1331  231 = 1100 Ans : (1100)
  11  14  1  
= 17 22  14    

  17  28. In the sum of the first 15828 prime numbers there is one
= 17 (308 + 9) = 17  317 even number and 15827 odd numbers.
Hence, 317 the multiple of 17 is the least value of the number  The sum is odd. Choice (D)
required.
Hence the answer is 317. Choice (B) 29. The sum of first 10 natural numbers is 55. When 10
consecutive natural numbers are added the sum will be of
21. Let the number be 31k + 7, where k = 0 1, 2, …… the form (10k + 55) where, k is a natural number.
If (31k + 7) is divided by 25, the remainder is 6. Among the given options only 785 is of the above form.
 31k + 7  6 i.e. 31k +1 is divisible by 25. Choice (A)
The smallest value of ‘k’ for which (31k + 1) is divisible by 25
is k = 4 30. Y1 = (1)Y0 = y
 The required number is 31  4 + 7 = 131 Ans : (131) Y2 = (1)2Y1 = y
22. The greatest value of the divisor is given by Y3 = (1)3Y2 = y
HCF [(6155, – 5), (4935, – 15)] Y4 = (1)4Y3 = y
= HCF (6150, 4920) = 1230 Ans : (1230) The cycle repeats after every 4 terms.
(A)  Yn is positive when n is even and is not divisible by 4.
23. Weight of each part (B) Yn is not positive for all odd values of n(for n = 3, 7, etc.)
(C) is false, because (A) is true.
= HCF  5 1 lb,7 3 lb, 8 1 lb 
 4 4 5 
Choice (A)

Triumphant Institute of Management Education Pvt. Ltd. (T.I.M.E.) HO: 95B, 2nd Floor, Siddamsetty Complex, Secunderabad – 500 003.
Tel : 040–40088400 Fax : 040–27847334 email : [email protected] website : www.time4education.com SM1002162/8
1 1 1  75  2
31.     Now,  x  1  =   x  1   4
 x 
6 1
2 ( 5 ) ( 7 ) 2  (5 ) (7 )    x

1  1 1
=    = 4 2  4  12  2 3
2 5 7
2
1  1 1  and so on 1  1
Similarly 1
   
x2  2
  x    2 = (4)2  2 = 14
8 2 1 2 7 9 x  x
1 11 1  1
Finally,      x4  = 14  4  2 3 = 112 3 Choice (B)
16 2 1 2  15 17  x4
Value of the given expression.
36. Let the 600-digit number be N. The first positive integer
20  1  1 1 1 1 1 1 1 1 
=         .......    occurs once in N, the second occurs twice, the third thrice
3  2  5 7 7 9 9 11 15 17   and so on.
The number has 600 digits.
= 20  1   1  1  =
20 1 12 8
 Choice (A)  The last (positive) integer occurring in N is a two-digit
3  2   5 17  3 2 (5) (17) 17
number or part of a two-digit number.
Let us work out how many digits we would get if we go up to
32. 1, 1 + 2, 1 + 2 + 3, 1 + 2 + 3 + 4 …… are all triangular numbers. 25.
 Any triangular number would be of the form Total number of digits in the number up to 99…… 9 (nine
nn  1 times) = 1 + 2 + …9 = 45.
(i.e. 1 + 2 + 3 + 4 + …… + n)
2 Total number of digits in the number from 1010…..10 (ten times)
where n is a natural number. to 2525…….25 (twenty five times) = 2(10 + 11 + ….25) = 560.
nn  1 Total number of digits would be 605. We need to leave out
Now, let = = 903 the last 5 digits.
2 The number = (1223334444…) (2525…)2.
 n2 + n  1806 = 0  n2 + 43n  42n + 1806 = 0 There are twenty two 25’s in the second bracket above.
 n(n + 43)  42(n + 43) = 0 The last four digits of the number are 5252.
 (n  42)(n + 43) = 0  n = 42 or  43  Remainder of the number divided by 16 is equal to that of
But n =  43 is not possible. 5252 divided by 16. i.e., 4. Choice (A)
 n = 42
For the other numbers given, we find that on solving the 37. Y2  8Y = (X2  2X)2  8(X2  2X)
equations the values that n takes are not natural numbers. = (X2  2X) (X2  2X  8) = X(X  2) (X  4) (X + 2)
Hence, 903 is a triangular number. Choice (D) = (X  4) (X  2) X (X + 2)
i.e. The product of 4 consecutive even integers.
33. Let SN denote the sum of the first N natural numbers. The product of any 4 consecutive integers is divisible by 24.
S36 = 666, S37 = 703 When each of X  4, X  2, X and X + 2 is expressed as 2(an
 N = 36. The number which the student had added twice integer), Y2  8Y = 16 (Product of 4 consecutive integers).
by mistake = 700  666 = 34.  Y2  8Y is divisible by (16)(24) = 384 Choice (C)
The sum of its digits = 7. Ans : (7)
38. 324 = (18)2 = 22 34
34. Since the number is a three-digit number, 7, 8, 9 are ruled The sum of all the factors which are multiples of 3
out as 7! = 5040 which is a four digit number 8! and 9! are = (20 + 21 + 22) (31 + 32 + 33 + 34)
more than that. 6! is also ruled out, as 6! = 720, which would = 7(120) = 840. Ans : (840)
then require 7! which is not possible. Therefore, the three
numbers x, y and z can be chosen from 0, 1, 2, 3, 4 and 5. 39. X = {8, 14, 20, …….368,374}
Since it is a three-digit number, 5 has to be one of them, and 8 = 2 + 6 (1) and 374 = 2 + 6 (62)
since at most one 5 is allowed the number has to be below Therefore, there are 62 elements in X. If we form pairs like
5! + 4! + 3! = 150 and how it is obvious that 1 is in the (8,374), (14, 368), there will be 31 pairs. From each pair, if
hundreds place and 5 has to be present (but not in the tens we can choose only one number, the sum of no two numbers
place). Hence, the number is of the form 1Y5. Y may be 0, selected will be 382. Thus, we can choose at the most 31
2, 3 or 4. Clearly 145 = 1! + 4! + 5!. numbers so that the given condition is satisfied.
 541 – 145 = 396 Ans : (396) Ans : (31)

35. (i) x2  4x + 1 = 0 40. 838695 is divisible by 5. Sum of its digits is 42; it is divisible
Dividing both sides by x; by 3.
 838695 is divisible by 15.
1
x4+ =0 838695 = 15 (55913)
x 55913 is divisible by 11.
1 55913 = 11(5083) = 11(5100 – 17) = 11 (17) (300 – 1)
x+ =4
x = (11) (17) (299) = 11(17) (13) (23)
2 838695 = (11) (13) (15) (17) (23)
1  1 The five two-digit numbers are 11, 13, 15, 17, and 23.
Now, x2 + =  x   2
x2  x  Their sum is 79. Choice (A)
= (4)2  2 = 14 41. For N to be divisible by 8, N must be divisible by 4.
2 The last two digits of N must be 56, 76, 96 or 68 (last two
1 1 
Now, x4 +   x2    2 digits of N cannot be 88 since N has distinct digits).
4 
x  x2  If the last two digits of N are 56, the last three digits of N must
= (14)2  2 = 196  2 = 194 Ans : (194) be 656 or 856 (for N to be divisible by 8).
If the last two digits of N are 76, the last three digits of N must
(ii) x4 – 1 =  2 1  2
 x    x 
1 
 be 576, or 776 or 976.
x4  x2   x2  If the last two digits of N are 96, the last three digits of N are
=  x 2  1   x  1   x  1 
696 or 896.
  x   x  If the last two digits of N are 68, the last three digits of N are
 x2  
568, 768, or 968.

Triumphant Institute of Management Education Pvt. Ltd. (T.I.M.E.) HO: 95B, 2nd Floor, Siddamsetty Complex, Secunderabad – 500 003.
Tel : 040–40088400 Fax : 040–27847334 email : [email protected] website : www.time4education.com SM1002162/9
N has distinct digits.  The last three digits of N can be 856, 48. P = 1 + 2 + 6 + 24 + 120 + 720 + 5040 + 40320 + 9! + …..+ 60!
576, 976, 896, 568, 768 or 968. = 46233 + 9! + …..+60!
For any value that the last three digits can take, the Each of 9!, 10!, ……60! is divisible by 3. Also, 46233 is
thousands digit of N has two possible values. divisible by 3. P must be divisible by 3.
N has (2) (7) i.e. 14 values satisfying the given conditions. If P is a perfect cube, it is a perfect cube divisible by 3.
Ans : (14) Any perfect cube divisible by 3 must be divisible by 33 i.e. 27
46233 is not divisible by 27. Each of 9!, 10!......60! is divisible
42. Let x = 3p + 6q – 9r, y = 3p – 6q + 9r, z = –3p + 6q + 9r. by 27.
E = x3 + y3 + z3 – 3xyz where x + y + z = 3p + 6q + 9r 9! + 10! + …. + 60! is divisible by 27.
= 3(p + 2q + 3r) = 0. P is not divisible by 27.  P is divisible by 3 but not by 27.
As x + y + z = 0, x3 + y3 + z3 = 3xyz.  P is not a perfect cube.
E=0 A:P = 46233 + 9! + …….+60!
E is at least zero as well as at most zero. Choice (C) Each of 9!, 10!,…….,60! ends with 0. P ends with 3.
No positive integer ending with 3 is a perfect square.
43. Tn = n P is not a perfect square.
Tn + 1 = n + 1 P is neither a perfect square nor a perfect cube.
Tn + Tn + 1 = 2n + 1, which is always odd. Choice (D)
Tn + Tn + 1 is an odd perfect square.
n  150 49. The typist numbers 9 single digit pages (1 to 9), 90 two digit
 2n + 1  301 pages (10 to 99), 900 three digit pages (100 to 999), 201
Tn + Tn + 1 is an odd perfect square not excluding 301. four digit pages (1000 to 1200).
Tn + Tn + 1 can be 9, 25, 49, 81, 121, 169, 225 or 289. The typist would have pressed
(It cannot be 1 as 1 = 0 + 1 but 0 is not a natural number). –9 times to number all the 9 single digit pages,
Tn + Tn + 1 has eight values. Ans : (8) –180 times to number all the 90 two digit pages,
–2700 times to number all the 900 three digit pages,
44. The number is divisible by 9 and 4. The sum of the digits of –804 times to number all the 201 four digit pages.
7 5 4 3 2 9 9 p 6 = 45 + p. Number of times that the typist pressed the number keys
As the number is divisible by 9, 45 + p must be divisible by 9. = 9 + 180 + 2700 + 804 = 3693 . Ans : (3693)
 p = 0 or 9.
As the number is divisible by 4, p6 must be divisible by 9. This 50. Let the four consecutive even natural numbers be 2a, 2a + 2,
condition is satisfied only if p = 9,  p = 9. Ans : (9) 2a + 4, 2a + 6.
P = 2a (2a + 2) (2a + 4) (2a + 6) = 16a (a + 1) (a + 2) (a + 3)
L.C.M of numerators Q = P + 16 = 16[a(a + 1) (a + 2)(a + 3) + 1] = 16[(a 2 + 3a)
45. L.C.M of fractions = (a2 + 3a + 2) + 1]
H.C.F of denominators
= 16 (a2 + 3a + 1)2 = 16[a(a + 3) + 1]2.
Hence, L.C.M of
3 6
, and
16
is One of a and a + 3 must be even.  a (a + 3) must be even.
14 35 21  a (a + 3) + 1 must be odd.  [a (a + 3) + 1]2 must be odd
L.C.M of 3 6 16 48 Q
, and =  (1) i.e. is odd.
H.C.F of 14 35 21 7 16
H.C.F of numerators Also it is a perfect square. (1) is true
H.C.F of fractions = Q is an odd multiple of 16.  Q is not divisible by 32.
L.C.M of deno minators
Q is divisible by both 8 and 16. Ans : (3)
3 6 16
Hence, H.C.F of , and is
14 35 21 51. The product of all the factors of a positive integer N is
H.C.F of 3 6 16 1 N(N)/2, where (N) is the number of factors of N.
, and =  (2) Product of all the factors of N is N2.
L.C.M of 14 35 21 120
(N)
From (1) and (2),  =2
2
L.C.M of fraction 48 1
 = 1440; (N) = 4 i.e. N has 4 factors.
H.C.F of fraction 7 210
Any positive integer having 4 factors is either the cube
 1440 times the H.C.F = L.C.M of a prime number or the product of two distinct primes.
Hence answer is 1440. Choice (A) Suppose N is the cube of a prime number p.
The factors of N are 1, p, p2, p3.
46. We have to find how many three digit numbers from 100 to Sum of all the factors of N excluding N = 1 + p + p2
500 are divisible by 7 but not by 11. 1 + p + p2 = 57  p2 + p – 56 = 0  p = 7(∵ p > 0)
The least multiple of 7 exceeding 99 = 7(15) Suppose N is the product of the primes p1 and p2.
The largest multiple of 7 less than 500 = 7(71) Let p1 < p2.
The number of numbers divisible by 7 from 100 to 500 The factors of N are 1, p1, p2, p1p2.
= The number of numbers from 7(15) to 7(71), i.e. number of 1 + p1 + p2 = 57  p1 + p2 = 56
numbers from 15 to 71, i.e. 57. (p1, p2) = (3, 53), (13, 43) or (19, 37)
The number of numbers divisible by 7 but not 11 = the N has four values. Ans : (4)
number of numbers divisible by 7  (The number of numbers 52. As the remainders obtained are the same, N must divide
divisible by both 7 and 11, i.e. 77) 39276  38304 = 972.
There are 5 numbers from 100 to 500 divisible by 77[77(2), 972 = 1 (972) = 2 (486) = 3 (324) = 4 (243) = (6) (162) = (9) (108).
77(3), 77(4), 77(5), 77(6)]  N can be 972 or 486 or 324 or 243 or 162 or 108.
 52 numbers satisfy the given conditions. Ans : (52)  N has 6 possibilities. Choice (C)
47. Let N be abcd.
N exceeds dcba by M, where M is a multiple of 45 53. The divisor would be a factor of (64484  62767) i.e. 1717
M = abcd – dcba = 45k which is 17  101. Hence, the required three-digit number is
(1000a + 100b + 10c + d) – (1000d + 100c + 10b + a) = 45k 101. Choice (A)
999 (a – d) + 90(b – c) = 45k
90(b – c) is divisible by 45.  999 (a – d) must also be 54. Let P and Q be denoted by 10a + b and 10c + d respectively.
divisible by 45 i.e. by both 9 and 5  (10a + b) (10c + d) = (10b + a) (10d + c)
a – d must be divisible by 5 100ac + 10ad + 10bc + bd
 (a, d) can be (5, 0). Choice (D) = 100bd + 10ad + 10bc + ac  ac = bd

Triumphant Institute of Management Education Pvt. Ltd. (T.I.M.E.) HO: 95B, 2nd Floor, Siddamsetty Complex, Secunderabad – 500 003.
Tel : 040–40088400 Fax : 040–27847334 email : [email protected] website : www.time4education.com SM1002162/10
Let us assume a < c. (one of the digits has to be smaller) We see that B can be greater than A or equal to but not less
As ac is not prime, we have the following possibilities. than A. Choice (B)
(i) If a = 1, c = 4 or 6 or 8 or 9 then
ac = 4 or 6 or 8 or 9. 59. Let the four two-digit numbers be ab, ab + 1, ab + 2, ab + 3.
As a, b, c and d are distinct, ac = 4 and ac = 9 are not S = ab + (ab + 1) + (ab + 2) + (ab + 3) = 4ab + 6.
possible. S = 10(q) + 0 = 10q.
If ac = 6, b = 2 and d = 3 or vice versa. S is divisible by 10.  4ab + 6 ends with 0.
If ac = 8, b = 2 and d = 4 or vice versa.  4ab ends with 4.  ab ends with 1 or 6
(P, Q) could be (12, 63), (13, 62), (12, 84), (14, 82) or S = 10q = 4ab + 6
(63, 12), (62, 13) (84, 12), (82, 14) 10q = 4(10a + b) + 6 = 4(10a + 1) + 6 or 4(10a + 6) + 6
(ii) If a = 2, c = 3 or 4 10q = 40a + 10 or 40a + 30
q = 4a + 1 or 4a + 3.
 ac = 6 or 8
q is a perfect square. If b is 1, then 4a + 1 is a perfect square.
If ac = 6, b = 1 and d = 6 or vice versa.
If b is 6, then 4a + 3 is a perfect square.
If ac = 8, b = 1 and d = 8 or vice versa.
ab is two digit number  a  1
 (P, Q) could be (21, 36), (26,31), (21,48), (28, 41) or
(36, 21), (31, 26), (48, 21), (41, 28) 1  a  9.  5  4a + 1  37 and 7  4a + 3  39.
The perfect square values of 4a + 1 are 9 and 25.
(iii) If a = 3, ac will not be a single digit number. No perfect square value exists for 4a + 3.
 a  3 is not possible. b cannot be 6.  b must be 1.
 We have a total of 16 possibilities. Choice (B) ab has two possibilities.
 Two combinations exist.
55. HCF of 3p 5q+4 77 115 and 3q+5 5p+4 7x 11x is 3x 5x 7x 11x . We
Alternative Solution:
can say x  5 ------- (1)
The sum of 4 consecutive numbers is 10 times a square.
Consider the indices of 3 and 5. If between p and q + 5,
As the sum is 10 times square, it has to be 10, 40, 90, 160,
q + 5 is the one which is not greater, then between p + 4 and
250 or 360. (As all the 4 numbers are two-digit numbers, the
q + 4, q + 4 would be the one which is definitely smaller. As
sum has to be less than 400). The sum of 4 consecutive
the HCF = 3x 5x 7x 11x, q + 5 = x and q + 4 = x.
numbers a, a + 1, a + 2, a + 3 is of the form 4a + 6. It cannot
This is not possible.
be 40, 160 or 360.
∴ Between p and q + 5, p is the one which is not greater, i.e.,
 4a + 6 = 10, 90 or 250 and a = 1, 21 or 61.
p = x. Between q + 4 and p + 4, p + 4 cannot be the index
As the numbers have to be two-digited, there are only two
of 5 in the HCF.
possibilities.
(∵ that index is x and x = p).
21 + 22+ 23 + 24 = 90 and 61 + 62 + 63 + 64 = 250.
∴ q + 4 = x. As q  1, it follows that x = q + 4  5 ------- (2)
Ans : (2)
(1), (2)  x = 5
∴ p = 5. (∵ x = min (5, x)). p, q  1. 60. 452 = 2025 and 462 = 2116.
 p + 4, q + 4  5. There are 45 perfect squares up to 2050.
x = min(p + 4, q + 4)  x  5 – (2) 123 = 1728 and 132 = 2197.
From (1) and (2), x = 5. There are 12 perfect cubes up to 2050 (Of these 13, 43 and
Also x = min(p, q + 5) and q + 5  6. 93 are squares).
 Only possibility is p = 5. Ans : (5) Number of natural numbers up to 2050 which are either
56. Fermat’s last theorem states that there are no solutions in perfect squares or perfect cubes = 45 + 12 – 3 = 54.
positive integers for the equation ad + bd = cd for d ≥ 3. Number of natural numbers up to 2050 which are neither
perfect squares nor perfect cubes = 2050 – 54 = 1996.
Therefore, d has to be 2. There are many solutions for a, b
Choice (C)
and c. eg 32 + 42 + = 52, 52 + 122 = 132, 82 + 152 = 172 etc.
But we see that the minimum possible value of a or b is
Solutions for questions 61 to 75:
3 [for c, the minimum value is 5]
 d is less than the minimum of a and b. Choice (A)
61. It is given that x, y, z are three successive prime numbers.
57. (p # q) # (r # s) = 1 From statement 
 HCF p # q, r # s = 1 x  y = 6, y  z = 4
LCM p # q, r # s   x  z = 10
 LCM (p # q, r # s) = HCF (p # q, r # s)  p # q = r # s (, y, z) could be (29, 23, 19) or (53, 47, 43).
Choice (D) Statement  alone is not sufficient.
From statement , there is a limit given which is x < 60.
58. We have to compare x + y + x + y with 2x + 2y Independently this statement is not sufficient.
If the fractional part of x is less than 1/2, 2x = 2 x + 1 If we combine both statements, we get two possibilities i.e.,
If the fractional part of x is equal to or greater than 1/2 2x 19, 23, 29 and 43, 47, 53. Hence data is insufficient.
= 2 x + 2. Choice (D)
Let a be the greatest integer less than or equal to x and b be
62. From statement ,
the greatest integer less than or equal to y.
Let f denote a fraction less than 1/2 and g a proper fraction When k = 22  31  51 or 21  32  51 or 21 3152 the number
equal to or greater than 1/2. We tabulate the possibilities for of factors of k is 12.
So we can’t determine k uniquely.
x, y, A = 2x + 2y and B = x + y + x + y
From statement ,
When k = 22  51  31 or few of its multiples then k is a
A B 2x + 2y x + y + x + y multiple of 4. Again, we do not get a unique value of k.
Statement  alone is not sufficient.
(a + 1) + (b + 1) +
a + f b + f (2a + 1) + (2b + 1) Using both statements, k = 22  51  31 Choice (C)
(a + b + 1)
(a + 1) + (b + 1) + 63. From statement 
a + f b + g (2a + 1) + (2b + 2)
(a + b + 1) or (a + b + 2) The given expression is
(a + 1) + (b + 1) + 1 1  1  1
a + g b + f (2a + 2) + (2b + 1) 7  k      7  k 
(a + b + 1) or (a + b + 2) 2 3 6 2 2
a + g b + g (2a + 2) + (2b + 2)
(a + 1) + (b + 1) +  This sum is an integer only when k is odd. So statement 
(a + b + 2) alone is sufficient

Triumphant Institute of Management Education Pvt. Ltd. (T.I.M.E.) HO: 95B, 2nd Floor, Siddamsetty Complex, Secunderabad – 500 003.
Tel : 040–40088400 Fax : 040–27847334 email : [email protected] website : www.time4education.com SM1002162/11
1  11  From statement , x + y is odd so either x or y is odd and the
From statement  7 1    5  1  = 7     other is even. So we cannot answer the question. Hence
2 3 6 2 6 statement  alone is insufficient. Choice (A)
 It is an integer only when  is a multiple of 3. But we do 72. The given number is 810A4B6C. We need to find A + B + C.
not know whether  is a multiple of 3 or not. So we can’t From statement , as the number is a multiple of 5 and 8,
C = 0 and B is odd.
answer the question. Choice (A)
As the number is a multiple of 9, the possible values of A and
64. From statement , a = b. B are as listed below.
 a + b = 2b and ab = b2
b2 < 2b if b = 1 and b2 = 2b if b = 2 8 1 0 A 4 B 6 C
otherwise b2 > 2b
So statement  alone is insufficient – – – 7 – 1 – 0
From statement , a = 1 – – – 5 – 3 – 0
 a + b = b + 1 and ab = b – – – 3 – 5 – 0
As, b + 1 > b is definitely true, a + b > ab. – – – 1 – 7 – 0
Hence statement  alone is sufficient. Choice (A) – – – 8 – 9 – 0

From statement , both A and B are non prime. There are
65. From statement , N is a product of two different single digit many possibilities for A, B and C.
numbers and N > 70. Hence N is (9)(8) = 72. Combining both statements, (A, B, C) = (8, 9, 0)
Hence statement  alone is sufficient  A + B + C = 17 Choice (C)
From statement , we can say that the two-digit number
greater than 70 which is a product of 3 distinct primes is 73. X is a natural number greater than 189.
78 [as (2)(3)(13) = 78]. Hence statement  alone is sufficient. From statement , the number has only five multiples less
Choice (B) than 1000.
i.e., it is less than 200
66. From statement ,  X  {190, 191,…….199}
The units digit of x and x2 is the same. So the units digit of x  This statement alone does not determine X.
is 0, 1, 5, or 6. So x could be 10, 11, 15, 16, 20, 21, 25, 26, From statement , the number is odd and does not end in 5.
30, or 31.  X  {191, 193, 197, 199 -----------}
Statement  alone is not sufficient. From statement , x2 can  this statement alone is also not sufficient.
be 121, 484 or 676. So x is either 11 or 26. Combining statements  and ,
Statement  alone is also not sufficient. there are 4 possible values of X which are {191, 193, 197,
Using both statements also x can be 11 or 26. So we can’t 199} All these numbers are prime, so X is definitely a prime
answer the question. Choice (D) number. Choice (C)
67. The sum of (2a  b) and (2a + 5b  4c) is 4a + 4b  4c. 74. Let the three integers be a, b and c.
From statement , c < 0, while a, b > 0. We can conclude that Given, abc = 40  At least one of a, b, c is even.
4a + 4b – 4c is positive. But, this is not sufficient. Statement : a + b + c is odd. Possibilities: 1 odd or 3 odd.
From statement , if c > 0, the numbers a, b, c are a, a + 1, As at least one is even, exactly 1 is odd.
a + 2 and 4(a + b –c) is 4(a – 1), which may or may not be Statement : ab + bc + ca is odd. Possibilities: 2(of a, b, c)
divisible by 3. are odd or all (of a, b, c) are odd. As at least 1(a, b, c) is
If c < 0, the numbers a, b and c are a, a + 1, –a –2 and even, exactly 2 are odd.
4(a + b –c) = 4[a+(a + 1) + (a + 2)] which is always divisible by 3. We can answer the question from either statement.
 Statement  alone is not sufficient, but statements  and Choice (B)
, taken together are sufficient to say a, b, |c| are successive
numbers. Choice (C) 75. From statement , N is the smallest number that leaves a
remainder of 4 when divided by 12, 13 or 14. Such
68. Is N the HCF of two numbers X, Y. numbers are of the form (LCM of 12, 13, 14)
From statement , if N is the HCF it is definitely factor of x, k + 4 = 1092k + 4 and the smallest such number is 1096.
y. But there may be more common factors of x, y. Statement  alone is sufficient
  alone is insufficient. From statement , N is the smallest number of the form
From statement , let N = 5, x = 30 and y = 20. kLCM(16, 17) – 10 or 272k – 10, which is 262
 x  y = 2N = 10 and the HCF is 10 which is 2N. If N = 5, x So N = 262
= 25 and y = 15, x – y = 2N and HCF(x, y) = N Again this alone is sufficient. Hence each statement alone,
Hence  alone is insufficient. Even if  &  are used together, is sufficient to answer the question. Choice (B)
we cannot answer the question. Choice (D)
ab Chapter – 2
69. K  0  ab = K  (Numbers – )
99
Concept Review Questions
 The product of K and 0  ab is an integer if K is a multiple
of both 9 and 11. Solutions for questions 1 to 20:
Both Statements  and  are required to answer the
question. Choice (C) 1. an + bn is divisible by a + b when n is odd. Since 103 is odd,
11103 + 14103 is divisible by 11 + 14, i.e., 25. Choice (C)
70. From statement I, the possibilities for (x y, z) are (28, 3, 31),
(42, 5, 47). So, the value of x is not unique. So statement  2. 382n  112n = (382)n  (112)n = (1444)n  (121)n. This is always
alone is insufficient. divisible by 1444  121 = 1323. The greatest number which
From statement . LCM of y and z is 527. Here 527 can divides it among the choices is 1323. Choice (D)
be expressed as 17(31), both of which are prime. So x is
3. 3200 = 34  (50) As the index of the power of 3 is divisible by 4,
31  17 = 14.
3200 has the same units’ digit as 34 i.e. 1.
 statement  alone is sufficient. Choice (A)
4500 has an even index.
71. From statement  x and y are integers. If xy is odd, then both Its units’ digit is 6.
x and y are odd. Hence statement  alone is sufficient.  Units’ digit of (3200) (4500) is 6. Ans : (6)

Triumphant Institute of Management Education Pvt. Ltd. (T.I.M.E.) HO: 95B, 2nd Floor, Siddamsetty Complex, Secunderabad – 500 003.
Tel : 040–40088400 Fax : 040–27847334 email : [email protected] website : www.time4education.com SM1002162/12
4. 33n – 1 = (33)n – 1 = 27n – 1n 20. The only three-digit perfect square in the form A9B is 196.
If N is a natural number, aN – bN is divisible by a – b.  A + B = 7. Choice (A)
27n – 1n is divisible by 27 – 1 i.e. 26.
The given statement is true for all values of n. Exercise – 2(a)
Choice (C)
Solutions for questions 1 to 25:
5. 25n + 1 = (25)n + 1 = 32n + 1n
If N is a natural number, aN + bN is divisible by a + b when N 1. Given number is 248 × 740 × 448
is odd. 2n ends with 2, 4, 8 or 6.
32n + 1n is divisible by 32 + 1 i.e. 33.  248 ends with 6, since 48 is multiple of ‘4’.
The given statement is true for odd values of n. 7n ends with 7, 9, 3 or 1.
Choice (B)  740 ends with 1, since 40 is multiple of 4.
4n ends with 4 or 6.
6. Units digit of (13687)3265 is the same as units digit of 448 ends with 6, since 48 is multiple of 2.
73265 = 74 (816)+1  The units digit of
 Units digit of 73265 is the same as that of 71, i.e. 7. 248 × 740 × 448 is 6.
Ans : (7) (∵ 6 × 1 × 6 ends with 6) Ans : (6)

7. The remainder when any number is divided by 25 is the


2. The units digit of
remainder when the number formed by the last two digits of
74k is 1
that number (i.e., 69) is divided by 25 which is 19.
74k+1 is 7
Ans : (19)
74k+2 is 9 and 74k+3 is 3
192567 is of the form 4k + 3
8. The remainder, when any number is divided by 9, is the
 57867192567 would end with a 3.
remainder when the sum of its digits is divided by 9. In the
Also, the units digit of
given problem, the sum of the digits of the number
24k is 6
= 37. Remainder, when it is divided by 9, is 1. Ans : (1)
24k+1 is 2
24k+2 is 4 and 24k+3 is 8
9. The remainder of ab  1 divided by b when a and b are co-
876 is of the form 4k.
primes is 1. In the given problem, 18 and 19 are co-primes
 1452876 would end with a 6.
 the remainder is 1. Choice (A)
 The units digit would be 13  6
i.e. 7 (since 3 < 6, 6 is deducted from 13). Ans : (7)
10. Any 10 consecutive natural numbers have a multiple of
among them 5.
3. Let f(x) =5x3  2x2  ax – b = 0
The product ends with a 5. Ans : (5)
Since (x  1) and (x + 1) are factors of f(x),
11. The largest power of 5 in 14! is 2. f(1) = 0
 14! ends with 2 zeros. The tens digit is 0. Ans : (0) 52ab=0
 a + b = 3  (1)
12. When 10000 is divided by 19, the remainder is 6. and f(1) = 0
 10000  19 = 9981 is the largest 4 digit number which 52+ab=0
leaves a remainder of 6 when divided by 19.  a  b = 7  (2)
Choice (D) Solving (1) and (2), we get
a=5 Choice (B)
13. The remainder, when 1000 is divided by 36, is 28.
 1000  28 = 972 is the largest 3-digit number divisible by 36. 4. 2360 = (23)120
 972 + 36 = 1008 is the least 4-digit number divisible by 36. 7 = 23  1
 The least 4-digit number which leaves a remainder of 10 Remainder when (23)120 is divided by 23  1 is 1120 = 1.
= 1008 + 10 = 1018. Ans : (1018) (By remainder theorem). Ans : (1)

14. The successive division is shown below 5. When an is divided by a prime number p and n is a multiple
of p – 1 then the remainder would always be equal to 1.
Number / Quotient 192 27 13 3 Here, 50 = (11  1)  5
Division 7 2 4  350 
Remainder 3 1 1 Re m  1
 11 
 
The last remainder is 1. Ans : (1)
Alternate method:
15. pqr86 is an even number, but not a multiple of 4. It cannot powers of 3 Remainder
be a square. Choice (B) 3 3
9 9
16. Any perfect square ending with a 5 must have a tens digit of 27 5
2. As the tens digit of 2a4b75 is 7, it is not a perfect square. 81 4
Choice (B) 243 1
729 3
6 7 8 9 10
17. 1076 is not a perfect square while 5776 is a perfect square. 729(3 ), 3 , 3 , 3 , 3 leave the same remainders as those
Choice (C) of 3, 32, 33, 34, 35 respectively.
 3x   x5 
18. PQ1 = (10R  1)2 or (10R  9)2 where R is a single digit number. Re m   Re m 3 
 11   11 
PQ1 = 100R2  20R + 1 or 100R2  180R + 81    
Tens digit of L.H.S is Q.  350   5
Tens digit of R.H.S is even. Re m   Re m 3  = 1 Ans : (1)
 Q must be even.  11   11 
Choice (B)    
19. The only three-digit perfect squares in the form P6Q are 169 6. 213 + 243 is divisible by 21 + 24 i.e., 45
or 361 or 961. 223 + 233 is divisible by 22 + 23 i.e., 45
 P has to be 1 or 3, or 9, i.e. odd. Choice (B)  Y = 213 + 223 + 233 + 243 is divisible by 45.
Triumphant Institute of Management Education Pvt. Ltd. (T.I.M.E.) HO: 95B, 2nd Floor, Siddamsetty Complex, Secunderabad – 500 003.
Tel : 040–40088400 Fax : 040–27847334 email : [email protected] website : www.time4education.com SM1002162/13
Moreover, Y is even (there are two odd numbers in the leave a remainder of r when divided by 6(241) = 1446.
expression) We obtain r as shown below.
 Y is divisible by 90 or Rem Y/90 = 0 Ans : (0) Let X = 6 x = 241 y + 94
y=6z+2
7. M = 49, 51, 49, 51, 49, 51, …. 49, 51 (600 digits or [As 94 leaves a remainder of 4 when divided by 6, 241y (and
300 groups of 2 digits each. Of these 300 groups, there are hence y) leaves a remainder of 2]
150 groups of 49 and 150 groups of 51). Least z = 0.  Least y = 2
L – S = (51) (150) – (49) (150) = 2(150) = 300 241(2)  94 482  94 576
x=   = 96.  Least x = 6.96
P = Rem  300  = 98. Choice (C) 6 6 6
 101 
  The number x is of the form LCM (6,241) k + 6.96 = 6(241)
k + 576 (This family of numbers represents an AP whose
8. Let N = 676, 767,... (900 digits or 300 groups of 3 digits common difference is 1446).
each) N is one of the numbers of this AP
M = (676 + 767) 150 = 216, 450
N
P = 216 + 450 = 666  r = Rem = 576 Choice (B)
1446
 Rem  N   666. Ans : (666)
 
 999 
16. By Wilson's theorem, (P  1)! is of the form K1 P  1
347 340  7 7
An immediate corollary of this is that, (P  2)! is of the form
9. Rem 347  Re m
47
 Re m
47 K2P + 1
100 100 100  95! is of the form 97 K + 1. The required remainder is 1
472 = 2309 Ans : (1)
092 = 81
476 = (81)(09) = 29 17. By Wilson's theorem, (P  1)! is of the form KP  1
 477 = (47)(29) = 63 Choice (C)  46! = 47 K  1
 46! (47) (48) (49) (50) = (47 K  1) (47) (48) (49) (50)
10. Rem 781234 = Rem 781220+14 = Rem 7814
50!
Rem 782 = 84, Rem 842 = 56, Rem 562 = 36 Let N = = (47 K  1) (48) (49) (50)
Rem 7814 = Rem (788) (784) (782) = Rem (36) (56) (84) 47
= Rem (16) (84) = 44 Ans : (44) Consider the remainder when N is divided by 47
N
972
Rem  ( 1) (1)(2) (3)   6  41
2612
11. Rem 326  Re m . All powers of 26 (except the first) 47
100 100 50!
 Rem  (41) (47) = 1927
end in 76. Choice (D) 47 2
12. Let (1 + x + 2 x2)100 = a0 + a1 x + a2 x2 + .... + a200 x200 KN N
(Rem  K Re m ) Choice (D)
Setting x = 1, we get a0 + a1 + .... + a200 = 4100 = 2200 KD D
Choice (A)

13. We need Rem 2275/137. The divisor d is prime and the index 18. N = 1! + 2! + 3! + …. 100!
of the power (275) is of the form nd + 1 [275 = 2(137) + 1]. By 168 = 7(24)
Fermat's theorem, 2136 leaves a remainder of 1  N  =  1 2  6  = 9
Re m  Re m 
2275 = 23a 2272  Rem 2275 = 8 (1)2 = 8. Choice (D)  24   24 
N  1  2  6  24  120  720  = 5
14. Rem 32180/149 = Rem 2900/149. By Fermat's Theorem, Rem Re m  = Re m 
2148/149 = 1 7  7 
 Rem 26(148) = Rem 2888 = 1  N = 7p + 5 = 24q + 9. N and all the other numbers X which
Rem 2900 = Rem 2888 Rem 212 = Rem 212 = Rem 4096 X X
(In all such questions, i.e Rem an/P, we have to be careful have there two properties (Rem = 9 and Rem = 5)
24 7
and treat the index (n) and the power an separately. can be obtained as follows.
We divided 2900 by 149 but 900 by 148. 7p = 24q + 4 ------ (1)
 2r  148 k   r   3q = 7q1 + 3 (Consider the remainders of the different
Rem   = Rem  2 
 149   149  terms when they are divided by the smaller of the two
    coefficients in (1), viz 7. The LHS is a multiple of 7.  The
For the index, the periodicity is p  1. (and not p). RHS has to be a multiple. As the second term on the RHS
 We divided 900 by 148 (and not 149) when we wrote with leaves a remainder of 4, the first term (i.e 24q or equivalently
the power, we have to divide it by p (and not p  1) 3q) has to leave a remainder of 3.  q1 is a multiple of 3)
4096 The values of q1, q, p, 7p + 5 and 24q + 9 are tabulated
Rem = Rem 149 (27) + 73 = 73
149 below.
 32180 
 Rem    73 . Ans : (73) q1 q p 7p + 5 24q + 9
 149 
  0 1 4 33 33
3 8 28 201 201
15. We want Rem 241202/1446 Now 1446 = 2(723) = 2(3)(241)
24 6 15 52 369 369
Rem = 0  Rem 241202/2 = 0
2
We see that any such number X has the form 168k + 33.
24  N is also of the form 168k + 33.
Rem = 0  Rem 241202/3 = 0
3
Rem 241202/241 = Rem 241200(242)/241 Alternate method:
= Rem (24240)5 (242)/241 = 1(Rem 576/241) = 94
This is LCM model 3 168 = 7(24).
 6 x = 241 y + 94 For n  7, n! is divisible by 168.
y=6z+2 N
Let N = 241202. N leaves a remainder of 0 when divided by Rem depends only on the first 6 terms of N,
168
6 and 94 when divided by 241. Let us say all such numbers
Triumphant Institute of Management Education Pvt. Ltd. (T.I.M.E.) HO: 95B, 2nd Floor, Siddamsetty Complex, Secunderabad – 500 003.
Tel : 040–40088400 Fax : 040–27847334 email : [email protected] website : www.time4education.com SM1002162/14
i.e they form a cyclic pattern with cycle length 6.
 1!  2!  3!  4!  5!  6! 
Rem  
 Rem 10
57 3
= Rem 10 = 6 and Rem
450
= 2.
 168  7 7 7
 1  2  6  24  120  720  Rem
X
= 6 – 2 =4  Both ,  are true. Choice (C)
= Rem   7
 168 
873 24. Difference of 7P and P3 is divisible by 10 i.e. 7P and P3 have
= Rem = 33. Ans: (33) the same units digit.
168
7P is odd,  P3 is odd  P is odd.
19. N = 127127 ………….. 1271 ( a total of 202 digits) P = 4k1 + 1 or 4k2 + 3 where k1 and k2 are whole numbers.
N = 1 + 1270 + 1270000 + …… If P = 4k1 + 1, 7P has a units digit of 7.
= 1+ 1270 (1 + 103 + 106 + …….. 10198)  P3 should have a units digit of 7. This is possible only if
The given divisor is 143. Instead of that we can consider a the units digit of P is 3.
convenient multiple of 143, viz, 143(7) = 1001. If P = 4k1 + 1, P = 13, 33, 53, 73 or 93.
If P = 4k2 + 3, 7P has a units digit as that of 73 i.e. 3.
= Re m 1  127011 1......  1  = 1271
N
Rem
   P3 also has a units digit of 3. This is possible only if the
1001  1001  units digit of P is 7.
 N  =  1271  = 127. Choice (C) If P = 4k2 + 3, P = 7, 27, 47, 67 or 87.
Re m
 143 
 Re m 
   143  P has 10 values. Ans: (10)
25. 25615 = 1630 = (17 – 1)30 = 1730 + 30C1 (17)29 (–1) + 30C2(17)28
 1111  = 9.
20. Re m
 19   (–1)2 + …………… + 30C
29 (17) (–1)29 +
  30C (–1)30
30
1111 Except for the last two terms which are –30(17) and1, all the
Successive powers of 1111 (or Rem = 9) leave the terms in the expansion are divisible by 172.
19
25615 = 172k + 30 (17) (–1) + 1 = 172k – 509
following remainders 9, 5, 7, 6, 16, 11, 4, 17 and 1 when
= 172(k – 2) + 2(289) – 509 = 172k1 + 69
divided by 19. After that (viz after the 9th power), the pattern
 11112222  289( 25615 )
256 15
repeats. As 2222 = 9(246) + 8, Re m  = 17, viz  Rem = 172 Rem = 289(69)

 19 
 17 4 17 2
= 19941. Ans: (19941)
the 8th number in the sequence above. Choice (D)

60  586098  6097 58  6096 582  .......  5898 


Exercise – 2(b)

21. x = Solutions for questions 1 to 35:


6098  5898
= 2  (1 + some positive number) 1. Given,
x>2 Choice (D) 733  1431  630.
Units digit of 733 = 7
22. 214N + 7 + 310N + 5 – 7 = (27)2N + 1 + (35)2N + 1 – 7 Units digit of 1431 = units digit of 431 = 4
= 1282N + 1 + 2432N + 1 – 7 Units digit of 630 = 6.
2N + 1 is odd. Units digit of 733  1431  630 is the same as the units digit of
an + bn is always divisible by a + b when n is odd. (7  4  6)
As 2N + 1 is odd, 1282N + 1 + 2432N + 1 is divisible by Required units digit = 8 Ans : (8)
128 + 243 i.e. 371.
214N + 7+ 310N + 5 – 7 = 371k – 7 2. Given,
Remainder of 214N + 7 + 310N + 5 – 7 divided by 371 is –7. 344 + 131  56 + 34  46
The equivalent positive remainder is 364. Choice (C) Units digit of 344 = 1
Units digit of 131  56 = 6
23.  : X = 1057 – 450 = 10(1056 – 45) Units digit of 34  46 = 4
1056 = (11 – 1)56 = (11 – 1) (11 – 1) …………….. 56 times Units digit of (344 + 131  56 + 34  46) is the same as the
= 11k + (–1)56 = 11k + 1 units digit of (1 + 6 + 4)
1056 – 45 = 11k + 1 – 45 = 11(k – 4) Required units digit = 1 Ans : (1)
X = 10 (11 (k – 4))  X is divisible by 11.
 is true. 3. The right most non-zero digit of 705340 is the right most digit
: X = (7 + 3) (7 + 3) ………….. 57 times = 7k + 357 of 75340. The right most digit of 75340 is 1.
357 = (33)19 = 2719 = (28 – 1)19 = 28k1 + (–1)19 = 28k1 – 1 (As 75340 is divisible by 4, 75340 has the same units digit as
1057 = 7k + 28k1 – 1 = 7k2 – 1 that of 74).
X = 7k2 – 1 – (7 (63) + 9) = 7(k2 – 63) – 10  Required digit is 1. Choice (D)
X
Rem   = –10. 4. The last digit in 42k is 6 and that in 42k+1 is 4.
7 782 is of the form 2k.
The equivalent positive remainder is 4.  The last digit of 424782 is 6.
 is true. The last digit of 92k is 1 and that in 92k+1 is 9.
 The last digit of 179137 is 9.
Alternate Solution: (Since, 137 is of the form 2k+1).
n  The last digit of 424782 + 179137 is 6 + 9 i.e., 5.
Rem 10 = 10 if n is odd, and 1 if n is even.
11 Choice (C)
450
Rem = 10. 689
769
11 5. Rem 1576  Re m = 76. All powers of 76 end in
100 100
Rem 10  450 = 0.
57
76. Choice (D)
11
n
The remainders of successive powers of 10, when divided a
6. leaves the remainder 1 when n is even
by 7, show the following pattern : 3, 2, 6, 4, 5, 1; 3, 2, 6, 4, a 1
5, 1; . . . . . . .  Required remainder is 1. Ans : (1)
Triumphant Institute of Management Education Pvt. Ltd. (T.I.M.E.) HO: 95B, 2nd Floor, Siddamsetty Complex, Secunderabad – 500 003.
Tel : 040–40088400 Fax : 040–27847334 email : [email protected] website : www.time4education.com SM1002162/15
7. (1923+ 1723) is divisible by 19 + 17 viz 36 because 23 is an (∵ 938 has the form 100k + 01 = 25 (4k) + 01).
odd number.
Note:  means equivalent remainder.
The units digit of the remainder when (1923 + 1723) is
Choice (B)
divided by 36 is zero. Ans : (0)
210 210
8. (24)19 is divided by 24 – 1. 15. Rem 114 can be obtained from Rem 114
Let x = 24, f(x) = x19. When f(x) is divided by x – 1, the 25 100
remainder is f(1) 210
1410
f(1) = (1)19 = 1 Ans : (1) Rem 114  Re m
100 100
9. (26)11 is divisible by 26 – 1 142  96, 962  16, 162  56
Using a similar method as in the above solution the  148  56
remainder is (–1)11 = –1. 1410  148 (142)  56(96)  76
 1410 
  Re m  76  1 (∵ 1410 has the form 100k + 76,
A negative remainder is not possible. It is converted to a
positive remainder by adding the divisor. Hence the actual  Rem   25 
 25   
remainder is –1 + 65 = 64. Choice (D)  
i.e. 25 (4k) + 76).
10. Rem (2582/3) = 2 (or  1)
 114 210 
 Rem (2582801/3) = 2 (or  1)  Rem   1
 25 
Rem (2579/3) = 2  
 Rem (2579401/3) = 2
 Rem [(2582801  2579401)/3] = 0 Alternate method:
Ans : (0) 1410 = Tenth power of an even number.
The tenth power of any even number ends with 24 or 76.
11. 783 = 73 (780) = 73(74)20 = 73 (2400 + 1)20 Also 1410 ends with 6.
= 73(2400k + 1) = (73) (2400k) + 343 where k is a natural
number.  114 210 
 Rem   1 . Choice (C)
When this is divided by 20, the remainder is 3.  25 
Choice (C)  

12. To find the remainder of any number N (say) when divided 489
849
16. Rem 784  Re m
by 99..9 (n nines), we group the digits of N such that there 100 100
are n per group starting from the right and add up all the 842  56, 562  36, 362  96.
groups. Then we can apply this process as often as needed
 848  96  849  (96)(84)  64.
until we get a value less than the divisor. This value is the
remainder. n = 56, 78, 56, 78,....56, 78 (1000 digits or 500
Alternate method:
groups of 2 digits each. Of these 500 groups, there are 250
The tenth power of any even number must end with 24 or 76.
groups of 56 and 250 groups of 78).
Also 8410 ends with 6.  8410 ends with 76.
Let Rem  N  = M. 849 will end with 4. Let us say it ends with a4.
 
 99  (a4) (84) = 76
M = (56 + 78)250 = (134) (250) = 3,35,00 Tens digit of L.H.S = units digits of (a.4 + 4.8 + tens digits of
Since M  99, we repeat the process until we get the 4.4) = that of 4a + 3  4a + 3 ends with 7
remainder.  4a ends with 4
 Rem  N  = 3 + 35 + 00 = 38. Choice (C)  a ends with 1 or 6.
 
 99   a4 = 14 or 64.
But 849 must be divisible by 4.
13. To find the remainder of any number N (say) when divided  a4 cannot be 14.  It must be 64. Ans : (64)
by 10n + 1, we group the digits of N such that there are N
786
digits per group starting from the right. We then add up all 326
17. Rem 1532  Re m
the alternative groups starting with the last group. Let the 100 100
sum we obtain be called L. We add up all the alternative 322  24, 242  76  324  76
groups starting from the second last group. Let the sum we 326 = 324 (322)
obtain be called S. Then we apply this process as often as
 Rem 326  (76) (24)  24 Choice (B)
needed until we get a value less than the divisor. This value
71 73
is the remainder. 18. Rem   1, Rem =1
Let N = 468 468 ... 468 (333 digits 72 72
= 1000 M + 468 (where M = 468468.......330 digits)  Rem 71 + 73 = (1)72 + 172 = 2
72 72 Ans : (2)
M = 468,468, ... 468 (330 digits or 110 groups of 3 digits each
of these 110 groups, L as well as S is the sum of 55 groups). 91 95
P = (468) (55)  (468) (55) = 0 19. Rem   2 and Rem =2
31 31
M Rem (91 + 95 ) = Rem (2)150 + Rem 2150 = 2 Rem
150 150
 Rem =0
1001 2150
N = 2 Rem (25)30 = 2 (1) = 2.
Rem = 468 Ans : (468) Ans : (2)
1001
20. Let (2 + 3 x)75 = a0 + a1 x + ...... + a75 x75
1008
931000  8 938
14. Rem 793  Re m  Re m Setting x = 1, we get a0 + a1 + ...... + a75 = 575
100 100 100 Choice (C)
1008 1008
Also, Rem 793 can be obtained from Rem 793 21. We want Rem 6722/73
25 100
722 = 72 (10) + 2.
932  49, 492  01, 012  01 Rem 672/73 = 1
 93 8 
 Rem    Re m  01  1  Rem 6720/73 = 1 (∵ Fermat’s theorem)
 25   25 
     Rem 6722/73 = Rem 62/73 = 36
Choice (B)

Triumphant Institute of Management Education Pvt. Ltd. (T.I.M.E.) HO: 95B, 2 Floor, Siddamsetty Complex, Secunderabad – 500 003.
nd

Tel : 040–40088400 Fax : 040–27847334 email : [email protected] website : www.time4education.com SM1002162/16


22. 81225 = 3900 the divisor is the prime number 179. 26. 1742 is divisible by 13.
900 = 178 (5) + 10 Required remainder = Remainder of (1742 + 8) (1742 + 10)
225
3900 (3178 )5 (310 ) (1742 + 12) divided by 13 = Remainder of (8) (10) (12)
Let R = Rem 81  Re m  Re m divided by 13 = 11. Ans : (11)
179 179 179
10
= Rem 3  3333 
(3333333333)
179 27. Rem = Rem   = 5 ---------(1)
5 16  16 
Rem 3  Re m
243 = 64
179 179  3144   36   36 
2 Rem   = Rem  81  = Rem  (16k  1) 
R = Rem 64  Re m 4096 = 158 ∴ R = 158  16   16   16 
179 179
     
Ans : (158) (16k + 1)36 = (16k + 1) (16k + 1),………………… 36 times
= 16M + 136 = 16M + 1
23. By Wilson’s Theorem (P  1)! is of the form K1 P  1  (16k  1)36   144 
An immediate corollary of this is that, (P  2)! is of the form Rem   = 1 i.e. Rem  3  = 1 --------------(2)
 16   16 
K2 P + 1    
 101! is of the form 103 K + 1. The required remainder is 1.  Required remainder = 5 + 1 = 6. Choice (D)
Choice (B)
3300
24. 100 = 25(4) 28. Rem 31  ( 1)3300 1
32
 7349 
Re m   (–1)349  –1  3  3332 
 4  Rem   = 4
 
 32 
 7 349   
 = Re m ( 49)  7 
174
Re m 
31 3300
 3332 
 25   25  Rem  = 1 – 4 = –3
     32 
 
 49174   7   (–1)174 7 = 7 Equivalent positive remainder is 32 –3, viz 29.
 Re m 
 25  Re m 25  Choice (C)
   
 7349 is of the form 4x + 3 and 25y + 7. Numbers of the form 29. Every natural number is of the form 3k or 3k + 1 or 3k + 2
mx + a and ny + b (where (m, n) = 1) are of the form mnz + where k is a whole number.
c where c is obtained as shown below. (3k)2, when divided by 9, will leave a remainder of 0.
4x + 3 = 25y + 7  4x = 25y + 4  y = 4y1 (3k + 1)2 = 9k2 + 6k + 1
Values of y1, y, x and 4x + 3, 25y + 7 are tabulated below. k can be of the form 3k, or 3k1 + 1 or 3k1 + 2 where k1 is a
whole number.
y1 y x 4x + 3 25y + 7  6k + 1 can be 18k1 + 1 or 18k1 + 7 or 18k1 + 13.
0 0 1 7 7  (3k + 1)2 when divided by 9 leaves a remainder of 1 or
1 4 26 107 107 7 or 4.
Similarly, it can be shown that (3k + 2)2 leaves a remainder
2 8 51 207 207 of 1 or 7 or 4.
 The square of a natural number leaves a remainder of
 N is of the form 100k + 7 0 or 1 or 4 or 7. The sum of all the possible remainders is 12.
Ans : (12)
Alternate method:
Successive powers of any positive integer n, when divided by 30. We want Rem 101283/514. Now 514 = 2(257)
any divisor d leave remainders that are in a cyclic pattern.
Rem 10/2 = 0  Rem 101283/2 = 0
When d = 100, the remainders are simply the last two digits of 1283
105( 256 )  3 3
the powers. When n = 7 and d = 100, the pattern is very short. Rem 10  Re m = Rem 10
There are only 4 possible values for the last two digits of the 257 257 257
powers of 7 which are 07, 49, 43 and 01. 256
349 4(87 )  1 (∵ Rem 10 1 ) = 229
7
 Rem 7 = Rem 7 = Rem = 7. 257
100 100 100 Let N = 101283. N leaves a remainder of 0 when divided by
Choice (B) 2 and 229 when divided by 257. Let us say all such numbers
leave a remainder of r when divided by 2(257) or 514. We
25. N = 121 + 122 + 123 + …. 12100 obtain r as shown below.
 5  5  .....5
2  100 Let X = 2 x = 257 y + 229 ( y has to be odd)
N   y = 1  2 x = (257 + 229) = 486
Re m  = Re m 
7  7  In general, X = 2(257) K + 486. (This family of numbers
The remainders when power of 5 are divided by 7 are listed represents an AP whose common difference is 514). N is
below one of the numbers of this AP. The required remainder is
51 – 5 486.  r = 486. Choice (D)
52 – 4
31. By Wilson’s theorem, (P – 1)! is of the form Kp – 1.
53 – 6
54 – 2  22! = 23 K – 1
55 – 3  22! (23) (24) (25) = (23 K – 1) (23) (24) (25).
56 – 1 Let N =
25!
= (23 K – 1) (24) (25)
------- 23
57 – 5
N
Rem    –1 (1) (2) = – 2  21
The remainders show a cyclic pattern and the cycle length is
6.  23 
Each of the first 16 complete cycles contributes 0. There are
4 powers in the next (the 17th) cycle  N 
N 5462  Rem  
2   (21) (23) = 483

 Re m
KN N
 K Re m .
 Rem = Rem = 3. Ans : (3)  23   KD D
7 7
Choice (B)
Triumphant Institute of Management Education Pvt. Ltd. (T.I.M.E.) HO: 95B, 2nd Floor, Siddamsetty Complex, Secunderabad – 500 003.
Tel : 040–40088400 Fax : 040–27847334 email : [email protected] website : www.time4education.com SM1002162/17
32. N = 14 + 24 + 34 …. + 1004 (say) M is the sum of the 46 consecutive integers. We can ignore
N the first 45 of these (their sum would be a multiple of 9 and
Re m 
7 165 N
think of only 165. As Rem = 3, it follows that Rem = 3.
 14  2 4  3 4  ...1004  9 9
= Re m  Note: The sum of n consecutive numbers is a multiple of n,
 7 
  provided n is odd. If n is even, say n = 2m, then the sum of
n consecutive numbers leaves a remainder of m, when
 (14  ....  7 4 )  .....  (92 4  ....  98 4 )  99 4  100 4  divided by n. Choice (A)
= Rem  
 7 
Chapter – 3
= Re m 14(1  2  4  4  2  1  0)  1  2  = 3. Ans : (3) (Number Systems)
 7  Concept Review Questions
33. Let A = 111111 = 111(1001) = 111(7)(11)(13) Solutions for questions 1 to 15:
 A is divisible by 13.
N = 111…..1(a total of 363 digits) 1. We have 502 = 256 + 128 + 64 + 32 + 16 + 4 + 2
= 1000 M + 111 where M = 111…..1 (a total of 360 digits) = 2 8 + 27 + 26 + 25 + 24 + 22 + 21
N 111  (502)10 = (111110110)2 Choice (D)
Rem = Re m = 7. (  M is a multiple of 13)
13 13 2. (1000001)2 = 1 × 26 + 1 × 1 = 65 Ans : (65)
Choice (C)
3. 7 532
34. X = 40k1 + 1 and Y = 40k2 + 2 7 76  0
40k 1 4(10k )1 7 10  6
: 3X = 3 1 = 3 1
7 13
Units digits of 3N has a cycle of 4. 01
 Units digit of 3X is that of 31 i.e. 3.  (532)10 = (1360)7 Ans : (1360)
3X = 3(40k1 + 1) = 120k1 + 3
 Units digit of 3X is 3 4. 12 1463
3X and 3X have the same units digits. 12 121  11 (B)
 3X – 3X is divisible by 10. 12 10  1
 is true. 0  10 (A)
40 k  2 4(10 k ) 2
: 7Y = 7 2 = 7 2  (1463)10 = (A1B)12 Choice (C)
Units digit of 7N has a cycle of 4.
 Units digit of 7Y is same as that of 72 i.e. 9. 5. The largest 3-digit septenary number is 666.
7Y + 7 (Y + 1) = 10k + 9 + 7(40k2 + 3) = 10k + 280k2 + 30 Ans : (666)
7Y + 7(Y + 1) is divisible by 10. 6. 8 239
 is true. 8 29 - 7
3-5
Alternate Solution:
 (239)10 = (357)8 Choice (D)
The units digit of successive powers of 3 or 7 show a cyclic
pattern and the cycle length is 4. 7. (AEB)16 = A × 162 + E × 16 + BX1
For 3 and 7, the patterns are shown below. = 10 × 256 + 14 × 16 + 11 × 1 = (2795)10 Choice (B)
Units digit of n 3 7 8. Let N be the number and (ak ak  1 …. a4 a3 000)2
Units digit of n2 9 9 = ak. 2k + ak1. 2k1
Units digit of n3 7 3 + …………. + a4.24 + a3.23
+ 0.22 + 0.21 + 0.20
Units digit of n4 1 1
= ak.2k + ak1.2k1 +…..
+ a4.24 + a3.23
As X = 40P + 1, the units digit of 3X is 3 while 3X =120P+3.
Clearly N is divisible by 8. Among the choices only 48 is
3x – 3X is divisible by 10.
divisible by 8. Choice (D)
As Y = 40Q+2, the units digit of 7y is 9 while
7(Y + 1) = 7(40Q + 3) = 280Q + 21.  7Y + 7(Y + 1) is divisible by 9. 16 1734
10. 16 108 – 6
Both ,  are true. Choice (C) 16 6 –C
0 –6
35. N = 120121122 ………………… 165  (1734)10 = (6C6)16. Choice (D)
Let M = 120 + 121 + 122 + …………… + 165
N M 10. We have
Rem   = Rem   (243)6 = 2 × 62 + 4 × 6 + 3 × 1 = (99)10
9 9 (201)7 = 2 × 72 + 0 × 7 + 1 × 1 = (99)10
(165 ) (166 ) (119 ) (120 ) and (143)8 = 1 × 82 + 4 × 81 + 3 × 1 = (99)10
M= – = (165) (83) – (119) (60) Choice (D)
2 2
= [9(18) + 3] [(9 (9) + 2)] – [9 (13) + 2] [9 (6) + 6]
11. To express a number in base B, the digits we use are
M 0, 1, 2,…B – 1.
Rem   = –6. The equivalent positive remainder is 3. To express a number in binary, the digits we use are 0
9 and 1 Choice (C)
N
Rem   = 3. 12. In any number system in which the base is at least 11, the
9 numerical value of A is 10. In the duodecimal system, the
base is 12. The numerical value of A is 10. Ans : (10)
Alternate Solution:
Let N = 120121 ------------ 165 13. The binary representation of any multiple of 16 ends with
and M = 120+121+ --------- +165 0000. Choice (D)
Triumphant Institute of Management Education Pvt. Ltd. (T.I.M.E.) HO: 95B, 2nd Floor, Siddamsetty Complex, Secunderabad – 500 003.
Tel : 040–40088400 Fax : 040–27847334 email : [email protected] website : www.time4education.com SM1002162/18
14. The decimal equivalent of the binary number 1. 011 is A B
1(20) + 0(2-1) + 1(2-2) + 1(2-3)
1 1 3 m n m6 m4 n4 n6 AB
=1+0+   1  1.375 Choice (D) 2 3 64 512 81 729 False
4 8 8
2 0
15. (224)5 = 5 (2) + 5(2) + 5 (4) = 64. Its cube root is 4. 3 6 729 19, 683 1296 – False
3 7 729 – 2401 – False
(4)10 = (11)3. Ans : (11)
4 8 212 218 212 218 True
Exercise – 3(a)
Choice (D)
Solutions for questions 1 to 8:
Solutions for questions 9 and 10:
1. We have (374)8 = 3 × 82 + 7 × 81 + 4 × 1 = (252)1
9. We have, 400 = 256 + 128 + 16
Now, 252 = 27 + 26 + 25 + 24 + 23 + 22
 The minimum number of times he needs to use the
 (252)10 = (11111100)2 Choice (C)
machine is 3. Ans : (3)
2. (386)12 = (534)10
10. Instead of one 256, he can use the quantity 128 twice, this
(177)12 = (235)10
implies, 400 = 128 + 128 + 128 + 16
 (386)12 – (177)12 = (299)10
 The number of times he has to use the machine
299 Now is 4. Ans : (4)
Now, 12
12 24 –B
2–0 Solutions for questions 11 to 25:

11. Given, (115)n = (1001101)


(299)10 = (20B)12 Choice (B)
  n  1  5   9  1  1
3. We have (418)10 = (110100010)2  n 2 8
 The minimum number of bits required is 9 Ans : (9)
 n 2
n5


77
n 8
4. We have (A)16 = (10)10
8n2 + 8n + 40 = 77n
(110101111)2 = 1 + 2 + 4 + 8 + 32 + 128 + 256
8n2  69n + 40 = 0
= 15 + 32 + 128 + 256
8n2  64n  5n + 40 = 0
= 47 + 128 + 256 = 384 + 47
= (431)10 8n (n  8)  5 (n  8)
When (431)10 is divided by 10, then the remainder is 1. (8n  5) (n  8) = 0
Choice (B)  n = 8. Ans : (8)
12. We have (25)12 = (29)10
5. We have  (25)12 $ (17)10 = (29)10 $(17)10
57 = 32 + 16 + 8 + 1 = 25 + 24 + 23 + 20 = 5 (29) + 2 (17) + 2)10
 (57)10 = (111001)2 = (145 + 34 + 2) = (181)10 Choice (B)
Also,
0.140625  2 = 028125 13. (51)k = (5k + 1)10
0.28125  2 = 05625 (50)k + 2 = (5k + 10)10
0.5625  2 = 1125 Given,
0.125  2 = 025 GCD is (9)10 and LCM is (180)10.
0.25  2 = 050 We have, the product of 2 numbers is equal to product of
their LCM and GCD.
0.5 2 = 10
(5k + 1) (5k + 10) = 9  180  k = 7. Ans : (7)
 (57143251)10 is equal to (111001. 001001)2
Choice (B) 14. We have, (305)13 = 3  169 + 5 = (512)10
(305)7 = (152)10
6. We have So, out of the given three options, only (512)10, i.e.,
2 4 (305)13 is a perfect cube. Choice (A)
(13.24)5 = 1  5 + 3  5° + 
5 25
15. We have, (33)7 = (24)10 and
14 (28)9 = (26)10
=8+ = 8.56. Ans : (8.56)
25 Arithmetic mean = (25)10
(IC)13 = (13 + 12) = (25)10
7. Let n = (ak ak 1 ak  2 ………. a1 a0)16  The required radix is 13 Ans : (13)
Then, n = ak 16k + ak  1 16k  1 + …….. + a1 161 + a0 160
16. Let the base of the system be n. All numbers which appear
Concatenating n with ‘0’, we have
without a base are in base ten. The sum of the roots is 13.
(ak ak  1 ………. a1 a0 0)16
 a = 13. The product of the roots is 40.  (44)n = 4n + 4 = 40
= ak 16k + 1 + ak  1 16k ……… + a0 16 + 0
or n = 9. Hence, the base or radix of the number system
= 16 (ak.16k + ak 1 16k  1 …………… + a0)
is 9. Ans : (9)
= 16n Choice (A)
17. We have,(34)7 = (25)10
8. Let a, b, c be 7-digit, 8-digit and 9-digit numbers respectively (31)8 = (25)10
in base m.  (34)7  (31)8 = (625)10 = (441)12 Choice (A)
i.e., m6  a < m7
m7  b < m8 18. LCM (3, 4, 5, 7) = 420
m8  c < m9  420  2 = 418 is the required positive integer
Let x, y be a 5-digit number and a 6-digit number respectively Ans : (418)
in base n i.e., n4  x < n5 19. (314)5 = 3 (52) + 1 (51) + 4 (50)
n5  y < n6 = 75 + 5 + 4 = (84)10
 The range m6 to m9 is a subset of the range n4 to n6. (412)6 = 4 (62) + 1 (6) + 2 (60)
We tabulate these four values for the given options (all in = (144 + 6 + 2)10 = (152)10
base 10) (314)5 @ (412)6 = (84)10 @ (152)10
Triumphant Institute of Management Education Pvt. Ltd. (T.I.M.E.) HO: 95B, 2nd Floor, Siddamsetty Complex, Secunderabad – 500 003.
Tel : 040–40088400 Fax : 040–27847334 email : [email protected] website : www.time4education.com SM1002162/19
= 5(84) – 2(152) + 60 5. 12 123456
= 420 – 304 + 60 = 176 12 10288 - 0
12 857 - 4
7 176 12 71 - 5
7 25 - 1 5-B
3-4
(176)10 = (341)7 Choice (D)  (123456)10 = (5B540)12 Choice (B)

20. (23516)8 = (010 011 101 001 110)2 6. (101101)2 = 1 (25) + 0 (24) + 1(23) + 1 (22) + 0 (21) + 1 (20)
= (0010 0111 0100 1110)2 = 32 + 0 + 8 + 4 + 1
= (274E)16 Choice (C) = (45)10
(201)8 = 2 (82) + 0 (81) + 1 (80)
21. (21)8 = 2 (81) + 1 (80) = (17)10 = 128 + 0 + 1
(23)5 = 2 (51) + 3 (50) = 10 + 3 = (13)10 = (129)10
 f[(23)10, (21)8, (23)5] (453)10 = (453)10
= f[(23)10, (17)10, (13)10] (101101)2 + (201)8 + (453)10
= [3(23) + 2(17) – 13] = (45 + 129 + 453)10
= 69 + 34 - 13 = (627)10
= (90)10
9 627
9 69 - 6
2 90
7-6
2 45 - 0
2 22 - 1  (627)10 = (766)9 Choice (B)
2 11 - 0
2 5-1 7. (231)16 = 2 (162) + 3 (161) + 1 (160)
2 2-1 = 2 (256) + 48 + 1
1-0 = 512 + 49 = (561)10
(231)8 = 2 (82) + 3 (8) + 1 (80)
(90)10 = (1011010)2 = 2 (64) + 24 + 1
The choices are of the form 8a + 1, 2b, 6c + 5 and 14d +11 = 128 + 25
We have to consider only choice B. Choice (B) = (153)10
(231)16 – (231)8 = (561 – 153)10 = (408)10
22. (26)7 = 2 (7) + 6 (70) = 14 + 6 = (20)10
(104)6 = 1 (62) + 0 (61) + 4 = (40)10 11 408
(88)9 = 8 (9) + 8 (90) 11 37 - 1
= 72 + 8 = (80)10 3-4
(40)2 = 1600 = 80 (20)
(i.e.) the numbers satisfy the condition b2 = ac (408)10 = (341)11 Choice (D)
Hence they are in G.P Choice (B)
8. 110110
23. (2000)8 = 2  53 = 64  16 = (1024)10 = (32) 2 –10001
The square root of (2000)8 is (32)10. Ans : (32) 100101

24. We have, (325)8 = (213)10 (100101)2 = 32 + 4 + 1 = 37


(213)2 = (45369)10 When any number in any base is divided by the base, it
= (130471)8 Choice (A) leave a remainder which is equal to the units digit. For
example, choice (A) i.e (112)4 is of the form 4a + 2
25. Given (1002)n = (345)10 Similarly the other choices are of the form 7b + 5, 5c + 4 and
 2 + n3 = 345 4d + 1.
 n3 = 343  We have to consider only choice D. (211)4
n=7 Ans : (7) = 2(16) + 1(4) + 1 = 37. Choice (D)

Exercise – 3(b) 9. 0.7265625 x 2 = 1.4531250 1


0.4531250 x 2 = 0.906250 0
Solutions for questions 1 to 25: 0.90625 x 2 = 1.81250 1
0.8125 x2 = 1.6250 1
1. (3AC)13 = C (130) + A (131) + 3 (132) 0.625 x 2 = 1.250 1
= 12 (A) + 10 (13)+ 3 (169) 0.25 x 2 = 0.5 0
= 12 + 130 + 507 0.5 x 2 = 1.0 1
= (649)10 Ans : (649)
 (0.7265625)10 = (0.1011101)2 Choice (C)
2. (100 101 011)2 = [(100)2 (101)2 (011)]8 = (453)8
Ans : (453) 10. (110101.11011)2
= 1(25) + 1(24) + 0(23) + 1(22) + 0(2) + 1 + 1(2–1) + 1(2–2)
3. [1110011101]2 = [0011)2 (1001)2 (1101)2]16 + 0(2–3) + 1(2–4) + 1(2–5)
(append two zeros at left) = 32 + 16 + 4 + 1 + 0.5 + 0.25 + 0.0625 + 0.03125
= (39D)16 Choice (C) = (53.84375)10 Ans : (53.84375)

4. A number abc in base n is 11. (281)10 < 2n


c an2 + bn2 + c (281)10 = 256 + 25
If 0 is appended to the right most digit of the number then the = 28 + 25
number becomes (281)10 < 2n
an3 + bn2 + cn  (2 + 25)  2
8 n

= n (old number)  n  9
So the new number is n times the old number. Number of bits required = 9 bits.
Choice (C)

Triumphant Institute of Management Education Pvt. Ltd. (T.I.M.E.) HO: 95B, 2nd Floor, Siddamsetty Complex, Secunderabad – 500 003.
Tel : 040–40088400 Fax : 040–27847334 email : [email protected] website : www.time4education.com SM1002162/20
It can be noted that 256 = (100000000)2 and (111111111)2  k =  9, 7
= 511 is the largest 9 bit number. So any number that lies But, k cannot be negative.
between 256 and 511 would require a minimum of 9 bits to k=7 Ans : (7)
represent it in binary. Ans : (9)
21. We have, (62)8 = (50)10
12. The remainder when (abcde)10 is divided by 9 is equal to the (144)8 = 4 + 32 + 64 = (100)10
remainder when a + b + c + d + e is divided by 9. in general, and (226)8 = 6 + 16 + 128 = (150)10
the remainder when (abcde)n+1 is divided by n is equal to the  (62)8 and (144)8 and (226)10 are clearly in arithmetic
remainder when (a + b + c + d + e) is divided by n. progression. Choice (A)
 The required remainder is 3. Choice (C)
22. (310)4 = 3 (42) + 1 (41) + 0 (40)
13. (1331)8 = 1 (83) + 3 (82) + 3 (8) + 1 (80) = 48 + 4 + 0 = (52)10
= 512 + 192 + 24 + 1 = (729)10 (110)4 = 1 (42) + 1 (41) + 0 (40) = (20)10
(729 )10 = (27)10 4 52, 20
The choices are 12 + 6, 42 + 3, 26 + 1, 32 + 3. 13, 5
Choice (C)
 L.C.M of (310)4,(110)4 = 13 (20) = (260)10
42
14. (132)4 = 1 x + 3 x 4 + 2 x 40 The first 3 choices are of the form 5a + 1, 6b + 2 and 4c + 1
= 16 + 12 + 2 We need to consider only choice B i.e. (1112)6 = 216 + 36 +
= (30)10 6 + 2 = 260. Choice (B)
30102 = (900)10 23. We have, (A)16 = 10
The choices are of the form 7a + 2, 4b, 7c + 4, 4d + 2. (11)2 = 3
We need to consider only choices 2 and 3 (13)8 = 11
(10230)4 = 256 + 2(16) + 3(4) and f (x, y, z) = (x + 2y) (2y + 2) (z + x)
(2424)7 = 2(343) + 4(49) + 2(7) + 4 = (16)  (17)  (21) = (5712)10 Ans : (5712)
= 686 + 196 + 14 + 4 = 900. Choice (C)
24. (346)n = (1211)5
2
15. (10111001)2 = 1(27) + 0(26) + 1(25) + 1(24) + 1(23) + 0(22)  3n + 4n + 6 = 125 + 50 + 6
 3n + 4n – 175 = 0
2
+ 0(21) + 1(20)
 3n + 25n – 21n – 175 = 0
2
= 2 7 + 25 + 24 + 23 + 1
= (185)10  n(3n + 25) -7 (3n+25) = 0
(11110)2 = 1(24) + 1(23) + 1(22) + 1(2) + 0(20)  (n – 7) (3n + 25) = 0
= 16 + 8 + 4 + 2 = (30)10
 25
Remainder when (185)10 is divided by (30)10 is (5)10  n = 7 or
Choice (C) 3
As radix cannot be a fraction, n = 7
16. 11
7 235
(215) 8 7 33 - 4
4-5 Ans : (454)
+ (476) 8
25. As 2 and 9 are single digit numbers, their values are 2 and 9
(713) 8 respectively in all bases which are 10 or more. Their product
is 18 which is represented as (15)n i.e.
 (215)8 + (476)8 = (713)8 18 = 5 + n  n = 13.
(543)6 = 5(36) + 4(6) + 3 = 207
Note: that the above addition is carried on in the base-8
13 207
system. Choice (B)
13 15 - 12
17. (112)3 = 1(32) + 1(3) + 2(30) = 9 + 3 + 2 = (14)10 1-2
(115)5 = 1(52) + 1(5) + 1(50) = 25 + 5 + 1 = (31)10 (543)6 = (207)10 = (12C)13 Choice (B)
Product of the numbers = (14)10 (31)10 = (434)10
Chapter – 4
12 434
(Geometry)
12 36 - 2
3-0 Concept Review Questions
 (434)10 = (302)12 Ans : (302)
Solutions for questions 1 to 35:
18. Let the scale of the number be n
1. 62 + 82 = 102
 1654 = n3 + 6n2 + 5n + 4n0 = n3 + 6n2 + 5n + 4  (1)
From the choices substituting n = 7 in (1)  the triangle is right angle.
Hypotenuse
we get 676 which is a perfect square. Choice (B)  Circumradius =  5 cm Ans : (5)
2
19. (39)11 = 3 x 11 + 9 x 110
= 33 + 9 = (42)10
(62)9 = 6 x 9 + 2 = 54 + 2 = (56)10 2. 122 + 162 = 202
Arithmetic mean of (39)11 and (62)9  The triangle is a right-angled triangle.
=  42  56  =  98  = (49)10  The point of intersection of the perpendicular sides is the
 2  orthocentre.
 2 10  10
Given that (49)10 = (94)n  Sum of the distances from the orthocentre to the vertices
 49 = 9n + 4  n = 5
of the triangle = (0 + 12 + 16) cm = 28 cm. Ans : (28)
(32)4 + (21)5 = (14)10 + (11)10 = (25)10 = (100)5
1
Ans : (100) 3. The inradius of a triangle is always less than (smallest
2
20. Given, (125)k = (68)10 altitude in the triangle). In the given problem, as the smallest
 k2 + 2k + 5 = 68  k2 + 2k  63 = 0 altitude = 18 cm, the inradius has to be less than 9 cm.
 (k + 9) (k  7) = 0 Choice (C)

Triumphant Institute of Management Education Pvt. Ltd. (T.I.M.E.) HO: 95B, 2nd Floor, Siddamsetty Complex, Secunderabad – 500 003.
Tel : 040–40088400 Fax : 040–27847334 email : [email protected] website : www.time4education.com SM1002162/21
4. The larger part and the smaller part are in the ratio of 2 : 1.  XY2 = XZ2 + ZY2
Choice (C) YZ = XY 2  ZX2  26 2  24 2 cm  10 cm .
Ans : (10)
5. Inradius = 18 cm = 3 3 cm. Choice (A)
2 3 22. Only an isosceles trapezium is necessarily a cyclic
quadrilateral. Ans : (1)
6. Circumradius = 18 cm = 6 3 cm. Choice (C)
3 23. BAC = BDC = 50°
(Angles in the same segment are equal).
Areaof  ABC 24 In triangle ABC, ABC + BAC + BCA = 180°
7. Area of triangle GFB =  cm2= 4 cm2. ABC = 180°  (BAC + BCA)
6 6
= 180°  (50° + 45°) = 85°
Ans : (4)
Ans : (85)
AB BD
8. 
AC DC 24. Area = 3 3 42  24 3 cm2. Choice (A)
2
2
BD = AB DC   10 8 cm  6 2 cm. Choice (D)
AC 12 cm 3 25. Let R and H represent the radius and the height respectively
of the original cone.
1 50 Let r and h represent the radius and the height respectively
9. QlR = 90° + QPR = 90° + 115 Ans : (115)
2 2 of the frustum.
Then it follows that
10. Only in an obtuse angled triangle, r Hh
AB2 + AC2 < BC2  BAC > 90° 
R H
i.e. x > 90° Choice (C) 2 1
h= H r= R
11. 82> +62 42 3 3
 The triangle is obtuse angled. The height of the smaller cone =
H
 Its circumcentre lies outside the triangle. Choice (C) 3
As the smaller cone and the original cone are similar, the
12. Incentre. Choice (A) R
radius of the smaller cone will be .
3
PS PT
13. As ST is parallel to QR,  2
SQ TR 1 R H 1 2
 Required ratio =   : R H  1 : 27
2
PT = PS TR   8 6 cm  12 cm Ans : (12) 3  3  3 3
SQ 4 cm Choice (B)

14. As PQRS is a cyclic quadrilateral, 26. Reflex  POR = 300°


R = 180°  P = 130° 2PQR = Reflex POR
S = 180°  Q = 110° Choice (D) PQR = 150° Ans : (150)

15. QS = PS SR   32 18  cm  24 cm Ans : (24) 27. As XZ is the diameter of the circle,
XYZ = 90°.
In triangle XYZ, YXZ + XYZ + YZX = 180°
2
16. EF =  24   3 12  cm = 16.8 cm Ans : (16.8)
YXZ = 180°  (XYZ + YZX)
= 180°  (90° + 35°) = 55° Ans : (55)
 5 5 
28. As AC and BC are tangents to the circles,
17. As ST is parallel to QR, OAC = OBC = 90°
PS ST
 In quadrilateral OABC, OAC + ACB + OBC + AOB = 360°
PQ QR AOB = 360°  (OAC + ACB + OBC)
2 = 360°  (90° + 50° + 90°) = 130° Choice (A)
QR = PQ ST   16 3 cm  12 cm Choice (D)
PS 4 cm
29. As RS || TU, XZT = XNR  (1)

18. Number of diagonals =


1010  3  35 Ans : (35)
(Corresponding angles are equal).

2 As VW || XY, VMR = XNR  (2)


(Corresponding angles are equal)
19. Let the centre of the circle be O
and the chord be AB. As PQ || RS, VOP = VMR  (3)
Let OC be the perpendicular (Corresponding angles are equal)
line drawn from O to AB, O
OC2 + AC2 = OA2 From (1), (2) and (3), VOP = XZT = 130°
AC2 = OA2 – OC2 = (152 – 92)cm2 A C B Choice (B)
 AC = 12 cm
AB = 2AC = 24 cm Ans : (24) 30. The quadrilateral formed by joining the midpoints of another
quadrilateral is always a parallelogram.
20. POQ = 2PRQ Its area is always half the area of the outer quadrilateral.
POR In the given problem, the quadrilateral formed is a
PRQ =  50 Choice (C)
2 200
parallelogram of area cm2 i.e., 100 cm2.
2
21. As Z is a point on the circumference and XY is the diameter
Choice (A)
of the circle, XZY = 90°
Triumphant Institute of Management Education Pvt. Ltd. (T.I.M.E.) HO: 95B, 2nd Floor, Siddamsetty Complex, Secunderabad – 500 003.
Tel : 040–40088400 Fax : 040–27847334 email : [email protected] website : www.time4education.com SM1002162/22
31. To two non-intersecting and non-enclosing circles, two direct TU QS 4
common tangents and two transverse common tangents can and   (given )  (2)
UR SR 3
be drawn. Choice (D)
3
From (1), TU = PT = 6 cm
32. To two circles which touch each other externally, two direct 4
common tangents and one transverse common tangent can 3
be drawn. Ans : (3) From (2), UR = TU = 4.5 cm
4
33. To two circles which intersect each other, two direct common  PR = PT + TU + UR = 18.5 cm Ans : (18.5)
tangents can be drawn. No transverse common tangent can
be drawn. Ans : (2) n(n  3 )
5. = 20, where n is the number of sides.
2
34. A triangle which has its circumcentre on one its sides must (2n  4) 12
have its circumcentre as the midpoint of its longest side.  n = 8. Interior angle = (90°) = (90)
n 8
Such a triangle must be right angled. Choice (C)
= 135° Choice (D)
35. The incentre, the centroid and the circumcentre coincide,
since the triangle is equilateral. 6. Let, number of sides = n
The required area is 0. Ans : (0) n2
Now, (180) = 144
n
Exercise – 4(a)  180n  360 = 144n  36n = 360
n = 10 Ans : (10)
Solutions for questions 1 to 35:

1. (i) 1 = 3 (vertically opposite angles) 36


7. Circumradius = cm = 12 3 cm. Choice (C)
5 = 7 (vertically opposite angles) 3
2 = 4 (vertically opposite angles)
6 = 8 (vertically opposite angles) 8. Let the sides be a cm, b cm and c cm
4 = 6 and 3 = 5 (alternate angles) Let a < b < c.  a + b < 2c and a + b + c < 3c.
1 + 8 = 180° (sum of the exterior angles on the same
20
side of the transversal) ----- (1) As a + b + c = 20 it follows that c >
3 – 8 = 90° given 3
 c can be 7, 8 or 9 .
1 – 8 = 90° --- (2) {∵ 1 = 3}
The possible values are tabulated below
 solving equations (1) and (2),
we get 1 = 135° and 8 = 45° a b c
 1 = 3 = 5 = 7 = 135° 5 7 8
and 2 = 4 = 6 = 8 = 45° Choice (C) 3 8 9
4 7 9
(ii) The perpendicular distances between (l and m) and 5 6 7
AC 3 Choice (B)
(m and n) are in the ratio 3 : 4, so  .
BC 4
9. PAB = 1/2 BAC A
7
 AB = (12) cm = 28 cm {∵ AC = 12 cm} PAQ = 1/2 BAC  BAQ
3
= 1/2 BAC  (90°  ABC)
Ans : (28)

2. Since PUSR is a parallelogram BAC  2ABC  180


=
UPR = 50° 2
B Q P C
TPU = 180°  QPU = 180°  (QPR + UPR)
= 180°  (60° + 50°) = 70° Ans : (70) BAC  2ABC  (ABC  BAC  BCA )
=
2
3. AB = AC2  BC 2  ( 41) 2  (9) 2 A ABC  BCA 80  40
=  = 20° Choice (C)
2 2
1681  81  1600  40
D E 10. The three medians divide the triangle into six triangles of
DE = 3  (40) = 20 3 equal areas.
2 C Quadrilateral CQAG consists of two such triangles.
B
Choice (C) The required ratio is 2 : 6 or 1 : 3 Choice (B)

4. Let U be a point on PR such that SU is parallel to QT. 11. P


P

S
T
V U
U

Q R Q R
S T
In PQR,
As QT || SU,
PQR + QPR + QRP = 180
PT PV 4
=  (given)  (1) PQR + QRP = 180º  QPR = 144  (1)
TU VS 3 As QS = QT, QST = STQ  (2)

Triumphant Institute of Management Education Pvt. Ltd. (T.I.M.E.) HO: 95B, 2nd Floor, Siddamsetty Complex, Secunderabad – 500 003.
Tel : 040–40088400 Fax : 040–27847334 email : [email protected] website : www.time4education.com SM1002162/23
As RU = RT, RTU = RUT  (3) Choice (D)
In QST and RUT, 15. R
SQT + STQ + QST = 180º Q
URT + RTU + RUT = 180º
Adding the two equations above, we get S T P
360 144
STQ + RTU = 108
2 Given that, RS : QT = 5 : 3. Clearly PQT and PRS are
(From (1), (2) and (3)) PT QT 12 3
STU + STQ + RTU = 180º similar triangles.  =  =
(QR is a straight line) PS RS (12  ST ) 5
STU = 180º  108º = 72° Choice (C)  ST = 8 cm.  RS = 5 cm and QT = 3 cm.
Now, the length of the common tangent RQ
AB BD
12. (i) As per angle bisector theorem,  = (ST )2  (RS  QT )2 = 64  4 .
AC DC
AC 2
RQ = 2 15 cm.  QP = 3 15 cm.
 DC = BD = 16 (3) cm = 4 cm
AB 12 cm (i) Clearly RQTS is a trapezium.
BC = BD + DC = (3 + 4) cm = 7 cm Choice (D) 1
 Its area = (2 15 ) (5 + 3) = 8 15 cm2.
2

 
(ii)
1
(ii) Area of PQT = 3 15 (3) = 9 15 cm2.
2 2
Choice (B)

16.
A
P U Q
5
3 O

4 S R
D C T
B

AB = AC2  BC 2  5 2  3 2 cm = 4 cm Let U be the point on PQ such that OU  PQ


AC DC Consider OUQ and TRQ
Now,  OUQ = TRQ = 90º and URO = RQT
AB DB
As two pairs of angles of UQO and RQT are equal, the
5 x4
Let DB = x cm.    5x = 3x + 12  x = 6 third pair of angles must also be equal.
3 x  UQO is similar to RQT
Choice (D) QR QU
   k (say )  (1)
13. Given that TR OU
D, E and F are midpoints of BC, CA and AB Area of the rectangle PQRS = (PQ) (QR) = (2UQ)  (2OU) =
and P, Q and R are midpoints of EF, FD and DE 4k (OU)2
we know that, Area of ABC = 4 Area of DEF ( As O is the centre of the circle, PQ = 2UQ and QR = 2OU).
But area of ABC = 64 sq. units Area of the circle =  (OU2 (1 + k2))
4 Area of DEF = 64
Area of rec tan gle 2 5
64 Given 
Area of DEF = Area of circle 3
4
Area of DEF = 16 sq. units. 4k (OU )2 2 5
Area of DEF = 4 Area of PQR
4 Area of PQR = 16

 
 k 2  1 OU 2

3
 12k  2 5 k2 + 2 5

16  5 k 2  6k  5  0  ( 5 k  1) (k  5 ) = 0
Area of PQR = =4
4
k= 5 or 1
Area of PQR = 4 sq. units. Ans : (4) 5
As PQ > PS, UQ > OU and k = QU/OU > 1.
14. In PQR and PRS,
k= 5 Choice (C)
QPR = RPS
QRP = RSP
As two pairs of corresponding angles of PQR, PRS are 17.  A X B   D X C
equal, the third pair of angles must also be equal. A B
 PQR and PSR are similar. BX 25 5 25 x
 
Raito of corresponding sides of PQR and PSR
XD 49 7 35 x 35 x
X
QR PQ PR
=   Area of  A D X 49 x
SR PR PS
7 C
PR2 = (PS) (PQ) = (32) (18) cm2 = (25 x ) = 35 x D
5
PR = 3218 cm  24 cm
PQ Ratio of the areas of  AXD and trapezium ABCD
Ratio of perimeters of PQR and PRS = = 3 : 4. 35 x 35
PR = = Choice (A)
Ratio of perimeters of PQR and PRS = 3 : 4. 35 x  25 x  35 x  49x 144

Triumphant Institute of Management Education Pvt. Ltd. (T.I.M.E.) HO: 95B, 2nd Floor, Siddamsetty Complex, Secunderabad – 500 003.
Tel : 040–40088400 Fax : 040–27847334 email : [email protected] website : www.time4education.com SM1002162/24
Ans : (72)
18. A
23.
B C

D
E
B A C D
Given AB = 8, BC = 10
B F C AD = 16, CD = 12
Let B’A = x, AC’ = 10  x B’D = 16 + x
DF || AC and EF || AB.
BB’ = y
Since DF || AC, triangles DBF and ABC are similar
x2 + y2 = 64  (1)
(∵B is common to the two triangles. Corresponding angles (6 + x)2 + y2 = 144  (2)
are equal) BD2 + AC2 = (16 + x)2 + 2y2 + (10  x)2
Since EF || AB, triangles EFC and ABC are similar. = 2x2 + 2y2 + 12x + 356 = x2 + y2 + (6 + x)2 + y2 + 320
2
= 64 + 144 + 320 = 528 Ans : (528)
 BF 
Ratio of the areas of DBF and ABC is   .Ratio of the 24. In a parallelogram, the sum of the adjacent angles is 180°.
 BC 
 x + 20 + x – 40 = 180°  x = 100°
2
 CF  Opposite angles are equal.
areas of EFC and ABC is   . x + 20 = y + 10
 CB  100 + 20 = y + 10  y = 110° Ans : (110)
2
 CF  64 4 CF 2 BF 3 AC2 + BD2 = 2(AB2 + BC2).
 CB  = .= ∴  .∴  . 25.
  400 25 CB 5 BC 5  2(AB2 + BC2) = 102 + 122 sq. units
and AB2 + BC2 = 122 sq.units. Choice (C)
9
∴ Ratio of the areas of DBF and ABC is .
25 26. (i) CDX = ABC = 130°
∴ Area of DBF is 144. (exterior angle of a cyclic quadrilateral is equal to the
opposite interior angle)
Area of ADFE = 400 – (144 + 64) = 192 Ans : (192) Since ABCE is a cyclic quadrilateral,
ABC + AEC = 180°
19. In triangles PAB and PQR, PAB = PQR and PBA = PRQ 130° + AEC = 180°  AEC = 50° Ans : (50)
(corresponding angles) and QAR is common.
Since the ratio of areas is 1 : 4, ratio of corresponding sides (ii) SOR = 180° – (SOR + ORS) = 180° – 2(20)°
is 1 : 4 = 1 : 2 = 140° (∵ OR = OS)
 Perimeter of PAB will be half of the perimeter of triangle  SQR =
1
SOR = 70°
PQR i.e., 12 cm. 2
Ans : (12) PTS = SQR = 70° (Exterior angle of a cyclic
quadrilateral equals the interior angle at the opposite
20. Let PT  QR . All lengths are in cm. vertex). Ans : (70)
(i) In PQT,
P
(PT)2 = 676  (QT)2  (1) 180  60
27. PTR = PRT = = 60°
(ii) In PTS, 2
(PT)2 = 625  (TS)2  (2) 26 TSR = PRT = 60° (alternate segment theorem)
26
(iii) In PTR, Choice (B)
(PT)2 = 676  (TS + 3)2  (3) 28. Let the radius of each circle be denoted by r cm.
From (2) and (3), 25 Perimeter of XYZ
625  (TS)2 = 676  (TS + 3)2 = XC + CD + DY + YF + FE + EZ + ZB + BA + AX
625 = 676  9  6(TS)  TS = 7. Q R
T S = (XC + CD + CD + YD  CD) + (YF + FE + FE + EZ  FE) +
 TS + SR = 10 cm.  TR = 10. (ZB + BA + BA + AX – AB)
But QT = TR = 10 ( ABC is an isosceles triangle) = (2r  CD) + (2r  FE) + (2r  AB)
 QS = QT + TS = 17. Choice (B) = [6(30)  (6 + 18 + 12)] cm = 144 cm. Ans : (144)
A
21. As BD  AC, 29. The two parallel chords could be on either side of the centre of
BD2 = AD. DC 5 the circle or on the same side of the centre of the circle.
 BD = 30 . Case : (See in Fig A)
D The two parallel chords lie on either side of the centre of the
 2 2 circle.
BC2 =  30  6  = 66 6
MN = MO + ON
 
MO = 20 (∵ AM = 15, OA = 25)
 30  + 5
2
B C
AB2 = 2 = 55 Choice (D) Fig: A
A M
A
22. AG =
2
12 = 8 and GE = 1 18  6 E 15
3 3
30° 25 20
Area of AGE G C
1 25 O
= (8 )( 6 ) sin 30° = 12
2 7
C
Area of triangle D 24 N
= 12(6) = 72

B Education Pvt. Ltd. (T.I.M.E.) HO: 95B, 2nd Floor, Siddamsetty Complex, Secunderabad – 500 003.
Triumphant Institute of Management
Tel : 040–40088400 Fax : 040–27847334 email : [email protected] website : www.time4education.com SM1002162/25
XP = XQ (= XR) = Circumradius of PQR
ON = 7 (∵ CN = 24, OC = 25) PQX = QPX = 35°(given)
 MN = 27 PXQ = 180° – (XPQ + XQP) = 180° – 2(35)° = 110°
Case : See Fig B Similarly, as PRX = 25°, it follows that PXR = 130°
Fig: B QXR = 360° – (PXQ + PXR) = 360° – 240° = 120°
A  PXQ < QXR < PXR
15 M Since PQ, QR and RP are chords of the circle circumscribing
13 PQR, PQ < QR < PR (∵ lesser the angle a chord subtends
24 N at centre, shorter will be the length of it)
C 7
O Alternatively:
MN = MO – NO = 20 – 7 = 13 Choice (B) In two isosceles triangles, say ABC, DEF. if AB = AC = DE
= DF, if A < D, then BC < EF. Triangles PXQ, QXR,
30. B PXR are isosceles. The equal sides of each of these are all
equal to the circumradius of PQR
The order of PQ, QR, PR is the same as the order of
PXQ, QXR, PXR
A C PQ < QR < PR
O E Perimeter of PQR is 24 cm
Both  and  follow (Perimeter > 3PQ and perimeter < 3PR).
Choice (C)
D
33. In a cyclic quadrilateral, if a, b, c and d are the lengths of the
If an equilateral triangle is inscribed in a circle, the centre of four consecutive sides and the diagonals are d1 and d2, then
the circle is the incentre (also the orthocentre, centroid and d1 d2 = ac + bd (Ptolemy’s theorem)
circumcentre) of the triangle. Hence, AC bisects A or
(10 )( 12)  (14 )( 15 )
BAO = 30° As ABC is an angle in a semicircle, ABC = 90°.  d2 = = 22 Ans : (22)
15
 Angles in ABC (and similarly ADC) are 30°, 60° and
90°, and the ratio of sides BC : AB : AC = 1 : 3 :2 34. Side of the polygon = 2(10) cos 75°
AB  BC  CD  DA
 The required ratio = 3 1
AC = 2(10) 15° 10
2 2
3  1 1 3
=  1 3 Choice (C) = 5( 6  2)
2 75°
Perimeter of polygon
= 60( 6  2 ) . Choice (B)
31. In PQR, PR2 = PQ2 + QR2
P S R
PR = 1602  1202  200 35. In the given triangle A
YY DC = EC = K1 (say)
Let PS = x X
E
SR = 200  x AE = AF = K2 (say) F
160 120
SQ2 = PQ2  PS2 = QR2  SR2 BF = BD = K3 (say)
= 1602  x2  2(K1 + K2 + K3)
Q
= 1202  (200  x)2
x = 128 = AB + BC + CA
SQ = 96, SR = 72 B D C
 K1 + K2 + K3 = 9. ….. (1)
Perimeter of PSQ = 384
If the inradius of PQS is a, K2 + K3 = AF + BF = AB = 5 ….. (2)
1
12896 (1) – (2) : K1 = 9  5 = 4
2  32
a=  DC = 4 cm Ans : (4)
384
2 Exercise – 4(b)
If the inradius of RSQ is b,
Solutions for questions 1 to 23:
1
7296
b= 2  24
288 1.
ℓ1
2 x°
XY2 = (a +  (a 
b)2 b)2
= 3136 + 64 = 3200 Ans : (3200) 75°

32.
P
2x°
ℓ2
By construction draw l3 parallel to l1 or l2,
a = x and b = 2x (alternate angles)
a + b = 75° but a +b = x + 2x =3x
X
3x = 75°  x = 25° Ans : (25)
Q R 2. PQY + PYQ = QPA (The exterior angle of a triangle is
equal to the sum of the interior angles opposite to it)
PQY = 120°  40° = 80°
X is a point inside ∆PQR which is equidistant from P, Q, R.
ZQR = 180°  80° = 100°
 X must be the circumcentre of PQR. BZX = RQZ + ZQR
Triumphant Institute of Management Education Pvt. Ltd. (T.I.M.E.) HO: 95B, 2nd Floor, Siddamsetty Complex, Secunderabad – 500 003.
Tel : 040–40088400 Fax : 040–27847334 email : [email protected] website : www.time4education.com SM1002162/26
= 100° + 25° = 125° Ans : (125) 42 cm < p < 66 cm Choice (D)
3. A + B + C = 180° QR
B – A = C – B 10. PQ = QT =
2
 2B = A + C
1
A + B + C = 3B As QTR = 90º and QT = QR, RQT = 60º.
B = 60° and hence 2
A + C = 120° 180  PQT
QPT = QTP =
A : C = 3 : 5 2
 A = 45° and C = 75° 180  90  60
C – A = 30°. Choice (C) =  15 Choice (A)
2
4. Let the side of the rhombus be a cm.
Let the longer and the shorter diagonals be ℓ cm and s cm 11. CAD = CDA = 20° (Since AC = CD)
respectively. BAD = 25° + 20° = 45°
a + l  s = 60 BCD = 360°  (40° + 45° + 20°) = 255°
2 BCT = 360°  (255° + 35°)
 2a + l + s = 120 = 360°  290° = 70° Ans: (70)
2a + l = 100
12.
 s = 20. Ans : (20) A
5. As AB is a chord and OM  AB 35°
AM = MB
PM2 – AM2 = (PM + AM) (PM – AM)
= (PM + MB) (PA) = (PB)(PA) 80° D
45° 100°
= 8(3) = cm2 = 24 cm2. B 25° E
Choice (C)
C
6. CRS + ESR = 180°
(∵ sum of interior angles on the same side of the transversal) Given that, EBC = 25°
BAC = 35° and AED = 80°
ESR = 180 – 50 = 130° {∵ CRS = 50°} (i) AED + AEB = 180°
ESR = STA = 130° (linear pair)  AEB = 100°
(∵ corresponding angles)
Ans : (130) (ii) In ABE,
ABE = 180°  (100 + 35°) = 45°.
7. Q  ABC
P = ABE + EBC = 70°.
80°
(iii) In AED,
Z EAD + ADE = 180°  80° = 100°.
S  [ABC + EAD + ADE] = 170°. Ans: (170)
T
M
120° 13. Given BC = 5 cm and BD : DC = 2 : 3
BD = 2 cm and DC = 3 cm.
R As AD  BC, AD = BD tan60° = 2 3 cm.

 
Join S and U
RSU = 180° 120° = 60°  2 2
 2 3  3  cm = 21 cm
2
AC2 = AD2 + DC2 = 2

QUT = PQR = 80°  


RUS = 180°  80° = 100°
SRU = 180°  (60° + 100°) = 20°  AC = 21 cm. Choice (D)
RQZ = 2  20° = 40° Choice (C)
14. B
A
8. If X is any point inside the triangle ABC, then AX + BX > AB,
BX + CX > BC and CX + AX > AC
By adding the three inequalities, we get E
2(AX + BX + CX) > AB + BC + CA
P
 (AX) + (BX) + (CX) > Choice (D) D C
2
AEB  DEC (AA Similarity)
9. A
area of ( A EB ) A B2

12 area of ( DEC ) DC 2

64 cm2 4
B    Area of (DEC) = 9 (64) cm2
21 C area of (DEC ) 9 4
In a triangle, the sum of two of the sides is greater than the = 144 cm2 Ans : (144)
third side. Also the third side is greater than the difference
between the first two sides.
(21 – 12) < AC < (21 + 12) 15. Required length = (1/2) (BE) = (1/2) 8 2  12 2
9 < AC < 33
= 2 13 Choice (A)
(21 + 12 + 9) cm < perimeter < (21 + 12 + 33) cm

Triumphant Institute of Management Education Pvt. Ltd. (T.I.M.E.) HO: 95B, 2nd Floor, Siddamsetty Complex, Secunderabad – 500 003.
Tel : 040–40088400 Fax : 040–27847334 email : [email protected] website : www.time4education.com SM1002162/27
16. Let, FB = x cm ar( ADE ) 1 A
CF = (15  x)cm 23. =
ar( ABC) 2
EF/AB = CF/BC
[Since ar(ABC) = ar(ADE)
15  x D E
 EF = (30) cm = (30  2x) cm + ar(trap.DECB) = 2ar(ADE)]
15 B C
2 1
Also, EF/CD = x/15  A D =  AD = AB/ 2
AB 2 2
x
 EF = (45) = 3x
15 = (12/ 2 ) cm = 6 2 cm
30  2x = 3x  x = 6 BD = AB  AD = (12  6 2 ) cm Choice (B)
EF = 3(6) = 18 cm Ans : (18)
Solutions for questions 24 and 25:
17. a2 + b2 + c2 = 50 ------- (1)
d2 + e2 + f2 = 50 ------- (2) AJ AJ
24. CK = CH  HK = AJ  IB = AJ 
ad + be + cf = 50 ------- (3) 2 2
Adding (1) and (2) and subtracting (2) (3) from the result, we AJ CK 3
get KD = HG =  tanCDK = 
(a – d)2 + (b – e)2 + (c – f)2 = 0 3 DK 2
 a = d, b = e and c = f CDK = tan–1  3  Choice (D)
 The two triangles are congruent. 2
 They will have the same perimeter and the same area.
Choice (C) 1
25. Area of ABIJ = (AJ + BI) (JI)
2
18. The ratio of the areas of two similar triangles is equal to the
ratio of the squares of the corresponding sides, (as 1 AJ  AJ AJ2
 AJ   
2  3
=
perimeters). 2 4
(32/40)2 = area of (ABC)/100 cm2
1
16 Area of EFHC = (CH + EF) HF
 area of ABC = (100) cm2 = 64 cm2 Ans : (64) 2
25
1
= (AJ + AJ) (HG + GF)
19. As per Apollonious theorem, 2
PQ2 + PR2 = 2(PS2 + QS2)  AJ AJ  2AJ2
1   
3 
 121  169  = (2AJ)
 QS2 = PQ  PR  PS 2 = 
2 2
 64  cm2 2  3 3
2  2  Required ratio = 3 : 8 Choice (C)
= 81 cm2
 QS = 9 cm Choice (B) Solutions for questions 26 to 28:

20. AD2 = AB2  BD2 26. Given AOB = 90°


A
2
= AB2  BC  AO = AB 2  OB 2 = 8 2  6 2 cm = 2 7cm
16
(Since BD = ¼ BC) In triangle AOD, as ADO = 45°, and AOD = 90°
Similarly, AD2 = AC2  CD2 DAO = 45°
2
= AC2  9BC B D C
16  AO = OD = 2 7cm

2 7 
2 2
 AB2  BC = AC2  9BC 2
16 16  AD = = 2 14 cm. Choice
 BC2 = 2(AC2  AB2) = 2 (212  92) cm2 = 720 cm2 (B)
 BC = 12 5 cm
Choice (A) 27.
B
A
21. AB2 = AE2 + BE2
AD2 = AE2 + ED2 A M P
AB2  AD2 = BE2  ED2
= (BE  ED) (BE + ED) D C
= (CE  ED) (BE + ED) (Since
BE = CE) = CD (BD)
ABCD is a square and BCP is an equilateral triangle.
BD = 65  63
2 2
cm2 B E D C
8 cm And PM || AB .
= 32 cm Ans : (32)
 PM is an altitude of the BCP.
22. Let BD = x cm
72 – (3 + x)2 = 52 – x2
Given that, AC = 22 2 cm.
 49 – (9 + x2 + 6x) = 25 – x2
 15 = 6x  x = 2.5 cm  BC = 22cm.  PM = 3 (BC) = 11 3 cm.
2
 AD = 5  2.5
2
  2
cm
 Required ratio, PM : AC = 11 3 : 22 2
= 2.5 3 cm
Choice (D) = 3 : 2 2 . Choice (B)

Triumphant Institute of Management Education Pvt. Ltd. (T.I.M.E.) HO: 95B, 2 Floor, Siddamsetty Complex, Secunderabad – 500 003.
nd

Tel : 040–40088400 Fax : 040–27847334 email : [email protected] website : www.time4education.com SM1002162/28


28. A 8 cm B

10 cm 10 cm

D 16 cm E C

Given that, AB = 8 cm,


BD = AC = 10 cm
CD = 16 cm and AB || CD .

 ABCD is an isosceles trapezium. If BE  CD , then ED =


4 cm. Since ABCD is an isosceles trapezium.

 BE = (BD)  (ED)
2 2
= 100  16 cm = 84 cm.

Now, BC = (BE)2  (CE )2 = 84  144 cm = 228 cm


= 2 57 cm  BC = AD = 2 57 cm. ( diagonals are equal
in an isosceles trapezium).
 BC + AD = 4 57 cm. Choice (A)

Solutions for questions 29 and 30:

29. Given OR = 5 cm and OP = 3 cm


As OPR = 90°, PR = 4 cm.
 AR = (8 + 4) = 12 cm.
In a circle when equal chords intersect, then the line
segments from the point to the circumference of the circle
which are adjacent to the angle made by the point with the
centre are equal.
As PR = PB, (In OPR and OQR, OR = OP, P = Q,
OP = OQ) it follows that AR = CR
CR = AR = 12 cm. Ans : (12)

30. As OP AR, OR  RC, and AR = RC,


OP = OQ, PR = RQ
 QC = AP = 8 cm Ans : (8)

Solutions for questions 31 to 35:

31. The perimeter of the triangle PQR


= PQ + PR + QR = PQ + PR + QM + RM
= PQ + PR + QB + RC = PB + PC = (8 + 8) = 16 cm.
Ans : (16)

Q 32.

40° 50°

O
40° 100° 80°
30°
T P R S

QPR = 180  QPT = 40º


As QR = PR, QPR = RQP = 40°
OQP = 90 (  PQ is tangent to the circle)
 OQR = OQP  RQP = 90° – 40° = 50°
In OQR, OQ = OR
 ORQ = OQR = 50°
QRS = QPR + PQR = 80º
ORS = QRS  ORQ = 80° – 50° = 30°
In ORS, OR = OS
 OSR = ORS = 30° and
ROS = 180º  (ORS + ROS) = 120°
Ans : (120)

Triumphant Institute of Management Education Pvt. Ltd. (T.I.M.E.) HO: 95B, 2 Floor, Siddamsetty Complex, Secunderabad – 500 003.
nd

Tel : 040–40088400 Fax : 040–27847334 email : [email protected] website : www.time4education.com SM1002162/29


Solutions for questions 38 to 46:
33. Let AD = x cm.
AB = AC = (x + 4) cm. S
In triangle ABD, AB2 = AD2 + BD2. 38. OA = 17 2  8 2 cm = 15 cm B R
 (x + 4 ) 2 = x2 + 52  x2 + 8x + 16 = x2 + 25.
OB = AB  15 cm = (22  15)cm 17
9 O
 x = = 1.125 Ans : (1.125) = 7 cm
8 17
 SR = 2BR = 2 17  7
2 2
cm P A 8
34. Join RP and PS Q
= 2 240 cm = 8 15 cm
RPS = 90° (∵ Angle in a semi circle). Choice (C)
RPQ > 90°.
Triangle RPQ is an obtuse angled triangle. 39.
RQ > RP and RQ > PQ. A R B
P P1

P
C

R Q RC is a common tangent to both the circles.


S
Now, AR = RC and BR = RC.
Let CAB = x and, CBA = y
Choice (A) ACR = x and BCR = y
Now, ACB + CAB + CBA = 180°
 (x + y) + x + y = 180°  x + y = 90°
35. A ACB = 90° Choice (B)
E
H 40. Let the number of angles which are x° be a.
Let the number of sides be N.
B Sum of all the angles = 180°(N – 2) = 360°  N = 4
O
D ax° + (4 – a) (180° – x°)
= Sum of all the exterior angles = a(180° – x°) + (4 – a)x
G (2x – 180) (2a – 4) = 0
F
C As x ≠ 90, a = 2. Ans : (2)
D
Let ‘O” be the centre of the circle. 41.
Let E, F, G, H be the respective points of contact of AB, BC,
CD, DA with the circle. C A
BE = BF, AE = AH, DH = DG and CG = CF E
Let EB = EF = x cm.
Given AB = 10 cm, AD = 3 cm and CD = 4 cm. 
O
 AE = AH = (10 – x) cm.
DH = DG = 3 – (10 – x) B
= (x – 7) cm.
 CG = CF = (1 – x) cm.
 BC = BF + FC = 11 cm. Choice (A) The perpendicular drawn from the centre of a circle to any
chord bisects the chord.
Solutions for questions 36 and 37: So CE = EA = 12 cm
7
As OEA is a right angled triangle, OE = cm
36. K is the centre of the circle. 2
DE = OD – OE =  25  7  cm = 9 cm
Let radius of the circle be r U Ans: (9)
 The diagonal of the square  2 2
= KT = r -M T
 The side of the square
42.
= KR = r . B
K S
2 R
 Ratio of the perimeter of the
square and circumference of  A
 r 
the circle = 4   : 2r 
 2
  C
22
= 2 : = 7 : 11 2 Choice (B)
2 7
D
AB 3
37. Ratio of the area of square and the area of the circle cos = =   = 30°
AC 2
 r 
2
1 BCD = 360° – 180° – 60° = 120°. Choice (B)
=   : r2 = :  = 7 : 44.
 2 2
  43. In a triangle ABC let D, E and F be the midpoints of BC,
Choice (C) CA and AB respectively. Then

Triumphant Institute of Management Education Pvt. Ltd. (T.I.M.E.) HO: 95B, 2nd Floor, Siddamsetty Complex, Secunderabad – 500 003.
Tel : 040–40088400 Fax : 040–27847334 email : [email protected] website : www.time4education.com SM1002162/30
2 Solutions for questions 47 and 48:
AB2 + AC2 = 2AD2 + BC  (1)
2
2
47.
28 E
BC2 + AC2 = 2CF2 + A B  (2) A 12 B
2
2
M G, H, I
AB2 + BC2 = 2BE2 + AC  (3)
2 9 41 10
Adding (1), (2), (3) we get 3(AB2 + BC2+ CA2) J, K, L
= 4(AD2 + BE2 + CF2)
D C
In any triangle, three times the sum of the squares of its sides AC = AB 2  BC 2 10 17 cm F
is equal to four times the sum of the squares of its medians.
4
Required sum = (36) cm2 = 48 cm2 Ans : (48) This is very close to the distance (41) between the 6 pairs of
3 points.
 Each of the three points in rectangle AEFD must lie close
44. P to one of the vertices A or D and each of the three points in
the other rectangle must lie close to the opposite vertex. i.e.,
C or B respectively.
 Choice (A) must be true. Choice (A)

48. From the above solution the points may lie on MA which is 1
S R Q
cm in length. All the points may lie on MA.
So, the distance between any pair of points among G, H, I is
PQ2 = PS2 + SQ2 = PS2 + (2QR)2 [∵ QR = RS] at most 1 cm.
= PS2 + 4QR2 From the choices GI can be 0.5 cm. Choice (A)
Also, PR2 = PS2 + SR2 = PS2 + QR2
PQ2  PR2 = 3QR2 Solutions for questions 49 to 55:
PR2 = PQ2  3QR2 = [202  3(8)2 ] cm2 = 208 cm2
49. r1 + r2 = 15  (1)
PR = 4 13 cm Choice (B)
r2 + r3 = 16  (2)
r3 + r1 = 17  (3) r2 B
45. Let D be the foot of the perpendicular drawn from A to BC. Adding (1), (2) and (3), we get;
r1 r2
A 2(r1 + r2 + r3) = 48
A
 r1 + r2 + r3 = 24 r1 r3
The radius of the largest
r3
circle is r3. C
r3 = 24  (r1 + r2)
= (24  15) cm = 9 cm
Ans : (9)
7
6 50.
E C

A  B
I J

B C G K L H
5

Let BD = x cm. F D
 62 – x2 = 72 – (5 – x)2
 36 – x2 = 49 – (25 + x2 – 10x)  10x = 12 AI = (1/4) AB = (1/4)(8) = 2
x = 1.2 cm IJ = AJ – AI = 2
Similarly, DL = LJ = 2
E is the mid point of BC  BE = 2.5 cm
Consider the right-angled triangle GJL. GJ = 4, JL = 2
DE = 1.3 cm Ans : (1.3)
 GL = 4  2 2 3
2 2
46. A
 GK = GL – KL = 2 3 – 2 = 2( 3 – 1) Choice (A)

51.
140°
S
120°

B C

The point S is the circumcentre of the triangle.

CSA = 360° – (140° + 120°) = 100°. A


2r 2r
CSA
ABC = = 50°. Ans: (50) 4r+2 3 r=a
2
r= a = a (2  3 ) Choice (B)
2 (2  3 ) 2

Triumphant Institute of Management Education Pvt. Ltd. (T.I.M.E.) HO: 95B, 2nd Floor, Siddamsetty Complex, Secunderabad – 500 003.
Tel : 040–40088400 Fax : 040–27847334 email : [email protected] website : www.time4education.com SM1002162/31
52.
S AC = 20 cm.
As we know the three sides of the triangle, we can find the
area of the triangle.
R  We can answer the question, using both the statements.
Choice (C)
Q
57. Given, BAC = 90°
 BC is the diameter of the circumcircle of the triangle
P T U V From statement , BC = 21cm
Hence AM would be the circumradius.
Let P = x. In PUQ, the external angle at Q is 2x AM = 1/2BC
(  P = QUP = x)  Statement  alone is sufficient.
In PSU, the external angle at U, i.e., SUV = 3x From statement , AD alone cannot yield the length of AM.
In PSV, the external angle at V, i.e., VSY = 4x Statement  alone is not sufficient.
In PRT, the external angle at T, i.e., RTX = 2x Choice (A)
58.
In PRV, the external angle at R, i.e., VRY A
= VSR = 3x
 VSR + VSY = 180°
5
 3x + 4x = 180°  x = 25 Choice (C) 33
7 D E

53. We know that the area of a quadrilateral is


1
= d1d2 sin C
2 B
F
1  3 As D, E, F are the mid points of AB, AC and BC respectively,
= (10)(14) sin60° = 70  = 35 3 sq.units.
2  2  the triangles AED, ECF, DFB and FDE are congruent.
   FCE = AED = 33°.

[  sin 180    sin ] Choice (C) From statement , FEC = 78°.
 BFE = FEC + ECF
54. In a cyclic quadrilateral the product of the diagonals is equal = 78 + 33 = 111o
to the sum of the products of the opposite sides. So, BFE = 111°
 (AC) (BD) = (AB) (CD) + (BC) (AD). Hence statement  alone is sufficient .
(AC) (BD) = 6(4) + 5(3) = 39 sq.units. From statement  BAC = 78°.
Choice (C) FEC = BAC = 78°.
 We can find BFE.
55. Hence statement  alone is sufficient. Choice (B)

D 59. Statement  alone is not sufficient, as it gives no lengths.


E Statement  alone is not sufficient, as we don’t know the
information about the other side or any of the angles of the
F triangle.
20° G From  and , we have
ABC = 60° and AB = 10 cm, AC = 10 cm.
For the sake of clarity, the circle has not been shown.  BCA = ABC = 60°
Let CE intersect BD at G  BAC = 180° – (60° + 60°) = 60°
BEG = BCE + EBC (∵ Exterior angle)  BC = 10 cm
1 We know all the sides, therefore, we can find the area.
= (BCA + DBC) Choice (C)
2
BEF ( ABD) is the complement of this angle. Hence
60. B
2ABD (or ABD + ACD) is the supplement of (DBC
+BCA). But together these angles make up the two interior
angles made by line CD and BA on the same side of the
transversal BC. As these angles are supplementary, CD is
parallel to AB. 10
 ACD = CAB = 20°
Ans : (20)
45º
A C
Solutions for questions 56 to 65: D E

56. As AD and BE are two of the medians, G must be the From statement , BCA = 45º. So triangle ABC is a right
centroid of the triangle. angled triangle, but we do not know the lengths of AC or BC,
 AG : GD = 2 : 1and BG : GE = 2 : 1. so statement  is not sufficient.
Statement  alone is not sufficient as it gives no lengths. From statement , we have BEA = 60º As triangle BDE is
Statement  alone is not sufficient, as it gives the information a right angled triangle, BD = BE sin(60°)
about only two sides
From  and , we have, AB = 10cm, BC = 20 cm. BD = 10 3 = 5 3 cm.
2
AG : GE = 2: 1,
BD
 GD = GE and BG = AG. AB = [   ABD is a right triangles].
Sin 45
 AD = BE.
In a triangle, if two medians are equal, then the sides as to  AB = 5 6 cm. Hence statement  alone is sufficient.
which these medians are drawn must be equal.
Choice (A)
 BC = AC

Triumphant Institute of Management Education Pvt. Ltd. (T.I.M.E.) HO: 95B, 2nd Floor, Siddamsetty Complex, Secunderabad – 500 003.
Tel : 040–40088400 Fax : 040–27847334 email : [email protected] website : www.time4education.com SM1002162/32
61.
P A Q

S R
C
From statement , the two circles have only one common
From statement , we do not know the information about the tangent therefore, one circle lies inside the other, which
point C, so we can’t answer the question. Statement  alone is means the distance between the centres is equal to the
not sufficient. difference between the radii. So statement  alone is
From statement , by knowing AC is parallel to PS we can’t sufficient.
answer the question as we don’t know the information about
B. Statement  alone is not sufficient.
Using both the statements,
2
we have AC = PS and PQ = AQ
3
1
 AC AQ
area of the triangle ABC 2 1
  
area of the rec tan gle PQRS PS  PQ 3
Using both the statements we can answer the question.
Choice (C) From statement , when two circles having only three
common tangents the circles touch each other externally.
62. A Therefore, the distance between centres is equal to the sum
of the radii. Statement  alone is also sufficient.
Choice (B)

P 65. F
O

E
D A
B
From the statement , by knowing the radius of circle we can’t
find the length of PO, because to find PO, we need OA as
well as AP. C B
Statement  alone is insufficient.
From statement , we have circumradius of triangle AOP as As ABD = BCF = 90°, AD and BF are diameters of the
14 cm. As OAP = 90°, the triangle AOP is a right angled circle.  E must be the centre of the circle.
triangle As OP is the hypotenuse it must be twice the  CEB = 2CFB and BEA = 2BDA.
circumradius.
[∵Angle at centre is twice the angle subtended by the arc at
Statement  alone is sufficient. Choice (A)
any point on the circumference of the circle].
63. A From statement , BDA = 30°.  BEA = 60°.
 Statement  alone is sufficient.
From statement , CFB = 30°.
B
r1  CEB = 60°
But we cannot find BEA.
r2
 Statement  alone is not sufficient.
Choice (A)
P
O1 O2
Chapter – 5
From statement , we have, (Mensuration)
r1 = 15 cm and r2 = 9 cm.
Also, O1 O2 = r1 + r2 = 24 cm. Concept Review Questions
The two triangles, AO1P and BO2P are similar.
Solutions for questions 1 to 36:
AO1 O1P 15 O2P  O1 O2
  .  =
BO2 O2P 9 O2P 1
1. Area of the triangle = (6) (8) sin30° = 12 cm2.
 5O2P = 3O2P + 3  24  O2P = 36 cm. 2
As O2BP = O2AP = 90° Ans : (12)
BP = O2P2  O2B2 2. Area of an isosceles triangle of base b cm and equal sides a
AP = O1P2  O2 A 2 b
cm each = 4a 2  b 2 .
As we know 4
O2P, O1P, O2B, O1A, we can find AP and BP, and thereby AB. In the given problem, b = 10 and a = 13.
Statement  alone is sufficient. 4132 10 2 = 60 cm2. Ans : (60)
10
Area of the triangle =
Statement  alone is not sufficient, as it does not have any 4
length. Choice (A)
3. Area of the triangle = rs. Choice (A)
64.

Triumphant Institute of Management Education Pvt. Ltd. (T.I.M.E.) HO: 95B, 2nd Floor, Siddamsetty Complex, Secunderabad – 500 003.
Tel : 040–40088400 Fax : 040–27847334 email : [email protected] website : www.time4education.com SM1002162/33

O
abc 18.
4. Area of the triangle = . Choice (C)
4R
C
6 6
5. Area of the triangle = 3
6 2 cm2 = 9 3 cm2.
4
Choice (D) 4
2
6. Let PS be the median from P to QR.
By Apollonius theorem, 2(PS2 + QS2) = PQ2 + PR2 A 2 4 B
2
3 4  6
2 2
PS2 =   cm2 = 3.5 cm2  PS = 3.5 cm.
Let A, B and C be the centres of the circles.
2 2
AB = 6 cm, BC = 10 cm and CA = 8 cm
Choice (C)
6  8  10
Semi perimeter(s) of the circle  cm  12 cm
7. AC and AE are secants. Let AD = X cm 2
(AB) (AC) = (AD) (AE) Area of the triangle  12(12  8)(12  6)(12  10) cm2
(2) (2 + 22) = (X) (X + 8)  X = 4  AD = 4 cm Ans : (4)
24 cm2.
8.
Note: We observe that the above triangle is a right triangle.
 Area = 1/2(6)(8) cm2 = 24 cm2. Ans : (24)
O
25 cm 19. Let the area of the square be 3 3 k and that of the triangle
7 cm be 4k
T S(S) = 3 3 k  S = (3 3 k )1/ 2
Let O be the centre of the circle and T be the point from which 1/ 2
the tangent is drawn.  16 k 
a  4k  a =  
 3 2
Length of the tangent = OT 2  Radius2 = 25 2  7 2 cm 4  3 
 
= 24 cm Ans : (24) Perimeter of the square = 4 (3 3k )1/ 2
9. Area = (6) (10) sin30° cm2 = 30 cm2. Ans : (30) (4 k )
Perimeter of the triangle  3
10. Area enclosed by a ring whose inner circle radius is r and ( 3 )1/ 2
outer circle is R is given by (R2  r2). (4 k )
In the given problem, R = 8 cm and r = 6 cm The required ratio is  4 (3 3k )1 / 2 : 3
 Required area = (82  62) cm2 = 28  cm2 = 88 cm2 ( 3 )1 / 2
Choice (B) =1:1 Choice (B)
1 20. Lateral Surface Area of the Prism = (Perimeter of the base)
11. Area = (6) (8) cm2 = 24 cm2. Ans : (24)
2 (Height) = 2 (4 + 2) (8) = 96 cm2. Ans : (96)

1 21. Lateral Surface Area = (4) (6) (10) = 240 cm2.


12. Area = (12 + 18) (15) cm2= 225 cm2. Choice (D) Total Surface Area = Lateral Surface Area + 2(Base Area)
2
= 240 + 2(6)2 = 312 cm2. Ans : (312)
13. Area of ABCD = Area of triangle ABC + Area of triangle ACD
22. Volume of the Prism = (Area of the base) (Height of the
=  1 4 12   1 6 12  cm2 = 60 cm2. Choice (C)
 
2 22  Prism) = 3
42 8  32 3 cubic cm. Ans : (32)
4

14. Perimeter = [(14) + 2(14)] cm ≈  22 14   214  cm 23. Volume = (12) (10) (9) = 1080 cubic cm. Ans : (1080)
 7 
 
= 72 cm. Ans : (72)
24. Longest diagonal = l2  b 2  h 2 Choice

72 cm2 = 38.5 cm2


90 (A)
15. Area of the sector AOB =
360
Ans : (38.5) 25. Lateral Surface Area = 2h (l + b) Choice (A)

16. Semi Perimeter 23 cm. 26. Let the length of the cuboid be l cm.
4l + 2l + (4) (2) = 44  l = 6 Ans : (6)
Area = 23  8 23  1023  1223 16 cm2
= 15 13 117 cm2  15015 cm2. Choice (A) 27. Total Surface Area = 2r2 + r2 = 3r2. Choice (C)

6 3 = 288 cubic cm.


17. Let the radius of the circle be r cm. 4
28. Volume = Choice (C)
Let the length and the breadth of the rectangle be ℓ cm and 3
b cm respectively.
 29. Let the radius and the height of the cylinder be r cm and h
[2(ℓ +b)] = 2r  ℓ + b = 2r
2 cm respectively.
If ℓ = r, b = r Volume of the cylinder = r2h cubic cm.
If b = r, ℓ = r 1
Volume of the cone = r2h cubic cm
In either case, ℓ = b = r 3
Required ratio = r2 : ℓb =  : 1 Choice (A)

Triumphant Institute of Management Education Pvt. Ltd. (T.I.M.E.) HO: 95B, 2nd Floor, Siddamsetty Complex, Secunderabad – 500 003.
Tel : 040–40088400 Fax : 040–27847334 email : [email protected] website : www.time4education.com SM1002162/34
2 Along each edge 10 cubes can be cut. (Along length, breadth
 rd of the cylinder is remaining. Choice (D) and height). Total number of small cubes that can be cut
3
= 10  10  10 = 1000 Ans : (1000)
30. Volume of the prism = (Area of the base) (Height)
1
Volume of the pyramid = (Area of the base) (Height) 40. The longest diagonal of the cuboid = l2  b 2  h2
3
 Ratio of the volumes of the prism and the pyramid
= 3 : 1. Ans : (3) = 15 2  20 2  25 2 cm  1250 cm ≈ 35 cm
Ans : (35)
31. Total Surface Area of the Pyramid
=  1 Perimeter of the baseSlant height Exercise – 5(a)
2 
Solutions for questions 1 to 35:
+ Areaof thebase
1
448 42 = 80 cm2. Ans : (80)
2 1. Let the sides of the triangle be a cm, b cm and c cm
Let a + b – c = 10, b + c – a = 20 and c + a – b = 30.
32. Total Surface Area = (6) (10 + 6) = 96 cm2. Adding these equations, we get a + b + c = 60.
Choice (C)
Area of the triangle = s(s  a ) (s  b ) (s  c )
33. Lateral Surface Area of the Frustum of a cone having top
radius of r cm, radius of the base of R cm and slant height of abc a b c aa b c ba b  c  c
      
ℓ cm = l(R + r) cm2. 2  2  2  2 
In the given problem, r = 8, R = 10 and l = 9
Lateral Surface Area = (9) (18) = 162 cm2. (a  b  c ) (b  c  a) (a  c  b) (a  b  c )
Choice (C) =
16
34. Total Surface Area = Lateral Surface Area + Top Area + (60) (10) (20) (30)
1 = = 150 cm2. Ans : (150)
Base Area = (Sum of the perimeters of the base and the 16
2
top) slant height + Top Area + Base Area
2. Third side = 58  (16 + 22) = 20 cm
1
= [4(6) + 4(10)8] + 62 + 102 = 392 cm2. Ans : (392) 16  22  20
2 Now, S = = 29
2
 Area of the triangle
35. Slant height = 5 2  122 = 13 cm Ans : (13) = 29( 29  16) ( 29  22) ( 29  20) = 154.11 cm2
Let the length of the altitude be h cm
1 = ½  22  h = 154.11
36. Volume = h (R2 + Rr + r2) Choice (B)  h = 14.01 m Choice (C)
3
3.
Solutions for questions 37 and 38: A

R is the base radius of the frustum, also of the cone from which 4 cm
the frustum is obtained.
Let H represent the height of this cone.
Then it follows that
r Hh B C

R H
Let AB = x + a and BC = x  a.
2 1 Area of the triangle ABC
As h = H , it follows that r = R
3 3 ( x  a)(x  a)
= x a .
2 2
=
H 2 2
The height of the smaller cone =
3 But area of the triangle is
As the smaller cone and the original cone are identical, the radius maximum.
R  a = 0.  AB = BC.
of the smaller cone will be .
3  AB = BC = 2 2 cm.
 Maximum possible area of the triangle is 4 cm2.
37. Ratio of the curved surface area = (Ratio of the radii) Ans : (4)
(Ratio of the slant heights) =  1   1   1 . Choice (C)
 3  3  4. Radius of the inscribed circle = 24
   9
2 3
Let each side of the inscribed triangle be ‘a’ cm
38. As ratio of the slant heights of the smaller and the original
Radius of the inscribed circle = circumradius of the inscribed
cone = 1 : 3, the ratio of the slant heights of the frustum and
triangle
the original cone = 2 : 3. Choice (D)
24 a
  a = 12
Solutions for questions 39 to 40: 2 3 3
39. Along one edge, the number of small cubes that can be 3 2 3
100  Required area = a = (12)2 cm2
cut  = 10 4 4
10
= 36 3 cm2 Choice (C)

Triumphant Institute of Management Education Pvt. Ltd. (T.I.M.E.) HO: 95B, 2nd Floor, Siddamsetty Complex, Secunderabad – 500 003.
Tel : 040–40088400 Fax : 040–27847334 email : [email protected] website : www.time4education.com SM1002162/35
Radius of bigger circle OB = 2 .  Area = 2
5. 67.2 = 3  22.4
89.6 = 4  22.4 Required ratio = 2 : 2 =  : 1 Choice (A)
112 = 5  22.4 10. Let the equal sides of the triangle be a each.
The given triangle is a right-angled triangle and hence the Perimeter of the triangle = 2a + a 2
hypotenuse of 112 cm is the diameter of the circle.
P
 Radius of the circumcircle = 1/2(112) cm = 56 cm 2a + a 2 = 8 2 + 8
Choice (C)
a=4 2
Let the radius of the largest
6. A X
possible quadrant cut out be r.
The hypotenuse of the triangle
is a tangent to the quadrant
at X.
E R
r = QX = Altitude drawn to the Q
G hypotenuse.
a
r= =4
2
B D C Area of the remaining region = Area of the triangle − Area of
the quadrant
Let AD and BE be the perpendicular medians.
22 ( 4 )2
1 2  r2 1
 
The point of intersection of any two medians of a triangle is 24
= a −  4 2 2
− (4)2 = .
the same. AD, BE and the median through C intersect at G, 2 4 2 7 (4) 7
2 Choice (B)
the centroid. As G trisects AD and BE, BG = (BE) = 6 and
3
1 11. Let the shaded areas in S1 and S2 be A1 and A2 respectively.
GD = AD = 4.
3 S2
The medians of a triangle divide the triangle into six triangles
S1 Q
of equal area.
Area of ABC = 6 (Area of BGD) S3
1 1
= 6 ( (BG) (GD)) = 6 ( (6) (4)) = 72. Ans : (72)
2 2

7. ½ d1 d2 = 21
d1 d2 = 42 P R
Also, side of the rhombus = 40/4 = 10 cm
2 2 Let the shaded and unshaded regions in S1 be a and b
d  d 
  1    2  = 102  d12 + d22 = 400 respectively.
 2   2  Let the shaded and unshaded regions in S2 be c and d
Now, (d1 + d2)2 = d12 + d22 + 2d1d2 respectively and let the area of ∆PQR be e.
= 400 + (2  42) = 484 = (22)2
As PQ2 + QR2 = PR2 (∵ Q = 90°)
 d1 + d2 = 22 Ans : (22)
S1 + S2 = S3 i.e. (a + b) + (c + d) = (b + e + d)
8. a+c=e
C As a + c = 30, e is also 30.
Choice (B)

AA B 12.
A

D E
D

The shaded area in the above figure is the area common to B O


the two circles.
Shaded Area = Area of sector ACD  Area of ACD + Area
of sector BCD  Area of BCD

= 2  90 22  1 22  = 2(  2) cm2. As BC = 2 cm and the radius of the circle is 1 cm, BC is the
 360 2  diameter of the circle.
Choice (B) Let O be the centre of the circle and D and E be the
respective points of intersection of AB and AC with the circle.
9. As OB = OD, and ABO = 60°, BDO is an equilateral
A E B triangle.
Similarly, ECO is an equilateral triangle.
As BOC = 180°, DOE = 60°.
H O F The triangles BDO, DOE and EOC are congruent.
The area of the shaded region
= Area of semi-circle– 3(Area of each equilateral triangle)
D C  
=  12  3 x 12 
G 3
 2 4 
Let OF = 1.  EF = 2 and area of square EFGH = 2

Triumphant Institute of Management Education Pvt. Ltd. (T.I.M.E.) HO: 95B, 2nd Floor, Siddamsetty Complex, Secunderabad – 500 003.
Tel : 040–40088400 Fax : 040–27847334 email : [email protected] website : www.time4education.com SM1002162/36
 3  AC
=  3 sq.cm. Choice (B) EC = = 15 m
2 4  2
 
BE = BC  EC
13. The cow can graze the shaded areas numbered  and . 2 2
= 20 m
1
Area of ABC = (20) (30) = 300 sq.m.
2
 1
Area of ADC = (18) (24) = 216 sq.m.
2
21
A Area of the plot = Area of ABC + Area of ADC = 516 sq.m.
14 Ans : (516)
7
18. X
 7
E D
Let the cow be tied at A.
22 270
Area of the region  =  21  21  = 10391/2 sq.m
7 360 2x 2x
22 90 F C
Area of the region  = 77 = 381/2 sq.m
7 360
 Total area that the cow can graze
= 10391/2 + 381/2 = 1078 sq.m
Ans : (1078) A M B
x
14. Let the arc length of 1st
sector be x cm. Arc lengths of 2nd,
3rd, 4th, 5th, 6th, 7th and 8th sectors are 2x cm, 4x cm, 8x cm, Let M be the midpoint of AB and XM = h
16x cm, 32x cm, 64x cm and 128x cm respectively. 2 2
Sum of the arc lengths of the sectors = 255x cm h2 + x  4 x 2  h2 = 15 x  h = 15
x
4 4 2
2 (1) 
 255x = x=
10 1275 Area of  ABX = 1 15 x 2 = 15 x2
2 2 4
If the central angle of the 1st sector is ,
  3 
21  Area of ABCDEF = 6   x 2 = 3 3 x2
 4 
2 1275   2

 Ratio of areas of ABX and ABCDEF = 15 . 2 = 5
1275 4 6
3 3
Choice (B)
Choice (C)
15.
ℓ/2 ℓ/2 19. Let AB = c; BC = a, AC = b
O P N A
b/2

b Q F
b/2
E
L ℓ M
(i) Area of the rectangle = ℓb.
b
(ii) Area of the triangle NPQ = B D C
8
b
(iii) Area of the triangle LMQ = 1 1 1
4 Area of ABC = bc sin A  ab sin C  ac sinB
2 2 2
 Area of the triangle PQL
Area of CDE = 1  2a   b  sin C = 
3b 2
= ℓb   b  b  b  = . Choice (D)
 8 4  8 2  3  3  9
 4 
2
Area of AFE = 1  2b  c  sin A = 
2  3  3  9
( 20)2  180 
16. Area of the pentagon = 5   cot   2
4  5  Similarly area of BDE =
9
= 500  1.376 = 688 cm2 Ans : (688) 
Area of DEF =   3 2  =
  3
 9 
17. AC2 = AD2 + CD2
/3
The ratio of the areas of DEF and ABC = =1:3
AC = 182  242 = 30 m 
Choice (C)
80  30
In ABC, AB = BC =  25 m
2 20.
Let BE be the perpendicular from B to AC.
BC2 = BE2 + EC2
As ABC is isosceles, A

.O
Triumphant Institute of Management Education Pvt. Ltd. (T.I.M.E.) HO: 95B, 2nd Floor, Siddamsetty Complex, Secunderabad – 500 003.
Tel : 040–40088400 Fax : 040–27847334 email : [email protected] website : www.time4education.com SM1002162/37
Rise in the level of the remaining plot
= Volume of the earth dug out = 562.5 m
Area of the remaining portion 1125
= 0.5 m. Choice (C)
Let the radius of the smaller circle (say D) be r. 25. Volume of the cube = (7)3 m3 = 343 m3.
The side of the smaller square (say T) = r + r = 2r Volume of the largest right circular cylinder = r2h
22
The radius of the bigger circle (say C) = 2r = (3.5)2 (7) cm3 = 269.50 cm3.
7
The side of the bigger square (say S)= 2 2 r  The volume of the metal which is not used
The area shaded by lines is (S – C) + (T – D). = 343  269.5 cm3 = 73.5 cm3. Ans : (73.5)
The area shaded by dots is (C – T).
26. Let the side of A as well as B be x cm.
The required ratio = 4r  r  8r  2r
2 2 2 2
Diameter of C = x cm
2r 2  4r 2
3
4  x  x 3
12r 2  3 r 2 34   
   cm3.
3  2 
Volume of C =
=

2 r 2  2r 2  = 2  2 Choice (D) 6
Let us say B is cut into n small cubes.
x3
21. The length and breadth of the cuboid formed are both equal Volume of each cube = cm3.
n
to (30  2y) cm each.
Volume of the cuboid = (30  2y) (30  2y) y x3
3
= 2(15  y) (15  y) (2y) Diameter of the sphere in each cube = cm.
n
The sum of 15  y, 15  y and 2y is constant. (i.e., 30)
Their product is maximum when 15  y = 15  y = 2y  
3
4  x3 
i.e., y = 5 Volume of each of these spheres =  n
 the volume of the cuboid is maximum, when y = 5. 3  n 
 
Choice (A)
= x cm3.
3

h 6n
22. Volume of the drum = (R2 + r2 + Rr)
Total volume of these spheres = x cm3.
3 3

22  5  6
  (242 + 152 + 24  15) = 42570/7 cu.ft Required ratio = 1 : 1.
7  3 
= Ans : (1)

Let the rise in the water level be H ft. 27. Let rate of flow per hour = x m/s
99(43 )H = 42570/7 Volume of water flowing through the pipe per second
 H = 42570  10  1 3 Choice (D) = x  25  25  x  m3
   
7(99)(43) 7 7  100  100  16 
23. Volume of water flown in 10 hours = 80(35)(2) = 5600 m3
 Volume of water flown into the tank per second
45 5600 7
= m3  m3
10(60 )(60 ) 45
3.5 x 7 7(16 ) 112
14 m   x= 
16 45 45 45
km 112  3600  24
The flow rate in is    8
hr 45  1000  25
Choice (A)

Height of the cone = Radius of the cone = 3.5 m 28. Number of bricks required = 0.9(1500 )(1000 )(800 )
Height of the cylindrical portion = (14  3.5) m = 10.5 m 10(8)(4)
Volume enclosed by the building (in m3) = 3,375,000
1  22 
Now cost of 3,375,000 bricks (in rupees)
22
= (3.5)2 (10.5) +   (3.5)2 (3.5)  400 
3  7 
= 3,375,000   = 13,500,000
7
= 4491/6 Choice (B)  100 
 The cost of the bricks is `one crore, thirty five lakhs.
24. Choice (B)
D C
 29. Q R
30 m 5

 5  O
A B Q1
50 m

(i) Area of the rectangular plot ABCD = 1500 m2.


(ii) Area of the trenches = (30)(5) + (50)(5)  (5)(5) m2.
= 375 m2.
(iii) Area of the remaining portion = 1125 m2.
(iv) Volume of the earth dug out = (375)(1.5) m3 .
P
Triumphant Institute of Management Education Pvt. Ltd. (T.I.M.E.) HO: 95B, 2nd Floor, Siddamsetty Complex, Secunderabad – 500 003.
Tel : 040–40088400 Fax : 040–27847334 email : [email protected] website : www.time4education.com SM1002162/38
82 + 152 = 172. volume of the larger box
 Maximum number =
 The triangle is right-angled. volume of each small box
By rotating the triangle about its hypotenuse, we get a double
cone. 30(20)(15) = 100. Ans : (100)
Volume of the figure generated = Volume of the double cone 6(5)(3)
= Volume of the upper cone (U) +Volume of the lower cone
(L) Exercise – 5(b)
Each of the two cones has a base diameter of QQ1. O is the
centre of the common circular base of either cone. Solutions for questions 1 to 45:
 QO = Q1O = Radius of the common circular base = r (say)
1. Let the radius of the circle be r units. Let the sides of the
QQ1  PR
square and the triangle be s units and a units respectively.
1 1
U = π(r2) (RO) and L = π(r2) (OP) 3 2
3 3 r2 = s2 = a
4
1 1
U+L= πr2 (RO + OP) = πr2(RP) r= s and a = 2s
3 3
 4
3
r = Length of the altitude drawn from Q to PR.
1 1 120 C = 2r = 2  s
Area of PQR = (8) (15) = 17r  r = .
2 2 17 S = 4s
2 T = 3a = 6s
1  120 
π   (17) 4
Volume of the figure generated = 3
3  17 
As  is less than 2, 2  s is less than 4s
4800 
= . Choice (A)  C < S.
17 Let us now compare S and T. Comparing S and T is
equivalent to comparing their fourth powers.
30. Let n be the number of small spheres formed S4 = 256 s
 n (volume of small sphere) = volume of big sphere T4 = 432 s
4  S4 < T4
 n 4 
 (2)(2)(2)  3 (30)( 30)( 30)
3  S<T
 n = 3375 Ans : (3375) C<S<T

31. The area covered by the lawn mower in one revolution is its Alternate method:
curved surface area. Let us consider regular polygons with n sides.
Area covered in 200 revolutions If we take a fixed length P and take increasing values of n
(3, 4, 5…) the area of the polygon keeps increasing with n.
 22  14  For a circle (n = ), this area is the maximum.
= 200(2rh) = 200(2)   ( 1) m2= 176 m2
 7  100  i.e., Area (Triangle) < Area (Square) < Area (Circle)
If we consider bigger triangle and squares so that their areas
Ans : (176)
are equal to the area of the circle (of circumference P), the
32. Let the length, the breadth and the height of the cuboid be perimeters will naturally be greater.
5x, 4x, and 3x respectively  If we consider regular polygon with equal areas, the
perimeter keeps decreasing with n and for a circle (n = ),
Longest rod’s length  (5x)2  (4x)2  (3x)2  10 2 cm
this perimeter (i.e. circumference) is the least.
50x2 = (10 2 )2 cm2  x = 2 cm T>S>C Choice (D)
Volume of the cuboid = (5x) (4x) (3x) = 480 cm3
Ans : (480) 2. Let the breath of the rectangle
be ‘b’ units.
33. Radius of the cone = 1/2 (12) cm = 6 cm  Diameter of the circle = b
Height of the cone = 12 cm  length of the rectangle = 2b.
1  22   Ratio of the area of rectangle
 Volume =  6 6 12  cm3
3  7 
and the circle
2
b
4 = (2b) (b) :    = 28 : 11. Choice (D)
= 452 cm3 Choice (A) 2
7

34. Let the length, breadth and the height of the cuboid be ℓ, b 3. The length of the pendulum is equal to the radius of the
 
and h respectively (all in cm). sector = 2r  = r =
214 = 2(42 + 35 + bh) 360 180
 bh = 30 ------- (1) 22 60
lb = 42 ------- (2)  r  = 44  r = 42 cm Ans : (42)
7 180
lh = 35 ------- (3)
From (2) and (3)
4. Volume of the water = (33  10  20) cu.m.
b/h = 6/5  b = 6/5 h
Area of the cross section of the sluice = 220 cm2.
6 2
Substituting in (1), we get h = 30  h = 5 220
5 = sq.m.
10000
6
 b = (5) = 6  Length of the water column in 5 hr = 33  10  20 = 300 km.
5 220
42 10000
l= =7 Ans : (7)
6 300
 Speed of the water flow = kmph = 60 kmph.
5
35. Each dimension (in m) of the larger box is a multiple of the
Ans : (60)
same dimension (in m) of each of the smaller boxes.

Triumphant Institute of Management Education Pvt. Ltd. (T.I.M.E.) HO: 95B, 2nd Floor, Siddamsetty Complex, Secunderabad – 500 003.
Tel : 040–40088400 Fax : 040–27847334 email : [email protected] website : www.time4education.com SM1002162/39
5. Let the other two sides be x cm and y cm  Required ratio = 3 Ans : (3)
x + y + 65 = 144  x + y = 79 ------ (1)
Also, x2 + y2 = 652 = 4225 ------ (2) 9. Let the radius of each circle be r cm.
Now, (x + y)2 = 792  (x2 + y2) + 2xy = 6241 2r = r2  r = 2
 2xy = 6241  4225 = 2036 Area of the shaded region = Area of square ABCD  Area of
4 sectors in it = (16  4) sq.units. Choice (B)
xy= ( x  y )2  4xy 10. Side of the innermost square = 4 cm.
Side of the nth outer square will be 2 cm more than the side
= 6241  2(2036 )  2209 = 47 ------ (3) of (n  1)th outer square to it.
Solving (1) and (3), we get  Side of the 9th outer square to it = 4 + 8(2) = 20 cm
x = 63, y = 16 Choice (A) Side of the 10th outer square to it = 4 + 9(2) = 22 cm.
Required area = 222  202 or 84 cm2. Ans : (84)
6. Let AB = AC = 16 cm and BC = 20 cm.
A
 AD = 10 cm 11. Let the circum radius be R and side of the polygon be a then
E a  10 
AD = ( AB)  (BD)
2 2 R = cos ec  R  cos ec
2 n 2 5
16
= 256  100 cm = 156 cm = 5  cosec 36 = 5  1.7 = 8.5 cm Ans : (8.5)
(AB)(CE) = (AD)(BC)
( AD)(BC ) 12. Angle covered by the minute hand in 25 minutes
 CE = = 25  6 = 150
AB B 10 D C
22
= 20 56  cm2
2.5 39 cm Choice (D)
 Required area =
7
(17.5)2  150 
 360 
 
6 cm = 4011/24 cm2 Choice (A)
7. Given that, PQ = 12 cm,
QR = 6 cm and SR = 18 cm. S
13. Length of the diagonal of the square = 5 2 units.
Let QU = x cm and TU = y cm.
Clearly PQR ~ TUR  Radius of the circumcircle = 5 units
TU UR 2
 =
PQ QR 5
Radius of the incircle = units
y 6x P 2
 = ------- (1)
5
12 6 If radius of the smaller circle is a, then +a+ 2a = 5
Also TUQ ~ SRQ. 2 2
TU QU y x 5 5
 =  = T 
SR QR 18 6 2 2
 y = 3x ---------- (2)  a
2 1
3x 6x
from (1) and (2)
12
=
6
5
 a 32 2
2
 
 3x = 12  2x
The required ratio
12 36
x= .  TU = y = . Q U R

   
5 5  5 
2 2 2
 :   5  : 4  5 3  2 2   1 : 1 : 3  2 2
2
1     
 Area of TQR = (6)  36  cm2 = 21.6 cm2. Ans : (21.6 )  2
  2 2  2 4
2  5 
 

8. Let AH = x, HG = 2x, GF = 3x = 2 : 1 : 4 (3 – 2 2 )2 Choice (D)


FE = 4x. EB = 5x
Let BC = h 14. As AB = 10 cm and the radius of the smaller circle is 5 cm,
Sum of the areas of p, q, r and s AB is the diameter of the smaller circle.
1  OA = OB = 10 cm.
= (sum of the bases of p, q, r and s) BC
2 2
1 [   AOB is a right angled isosceles triangle]
= (x + 2x + 3x + 4x)h = 5xh (where h = BC)
2 Area of the sector OAB
Area of the rectangle ABCD = (x + 2x + 3x + 4x + 5x)h = 15xh.
Required ratio = 3
=
90
360 
 
2
 5 2 cm2 
25 
2
cm2 .

Alternate method: Area of the shaded region


1
Area of ABC = (Area of rectangle ABCD)  25  
=  25    cm2= 25 cm2
2 
2 Choice (C)
 2
BCBE
Areaof  BCE 2

Areaof ABCD BCAB 15.
A P B
BE 5 1
=  
2AB 215  6
(From the given ratio)
 Sum of the areas of p, q, r and s
R Q
=  1  1  (Area of rectangle ABCD)
2 6
1 T
= (Area of rectangle ABCD)
3
D S C
Triumphant Institute of Management Education Pvt. Ltd. (T.I.M.E.) HO: 95B, 2nd Floor, Siddamsetty Complex, Secunderabad – 500 003.
Tel : 040–40088400 Fax : 040–27847334 email : [email protected] website : www.time4education.com SM1002162/40
16.
S R
Let RT = 1. Area of RTD = (1/2)(RT)(TD)
TS = 1 and SQ = 2, Area of QTS = (1/2) (TS) (QS)
Required ratio = 2 : 1 Choice (D) M N

P
Q

Given that, PQRS is a square and M is the mid point of PS.


Clearly N is the mid point of SQ, since N is the point of
intersection of diagonals.  MN || PQ .
Let the side of the square be ‘a’ units.
a a
 PQ = a, PM = and MN = .
2 2
1
(i) Area of the trapezium PMNQ = (PM) (PQ + MN)
2

=  3a  = 3a .
a 2

4  2  8

(ii) Area of the square = a2.


 3a 2 
 
 8 
(iii)  Required percentage =    100 = 37.5%.
a2
Ans : (37.5)

17. Le the length and the breadth of the rectangle be ℓ cm and b


cm respectively.
(ℓ + 3) (b + 3) = ℓb + 72  (1)
b(ℓ + 1) = ℓb + 9
b=9
Substituting b in (1), ℓ = 12 Ans : (12)

18. Let AB = ℓ, AD = b
2 (ℓ + b) = 68
 ℓ + b = 34 --------- (1)
ℓ=b+6 --------- (2)
Solving (1) and (2), we get
l = 20, b = 14
1
Now AP = (20) cm = 5 cm
4
2
AR = (14) cm = 4 cm
7
 Area of rectangle APQR = 5(4) cm2 = 20 cm2
 Area of QSR = ½ (Area of rectangle APQR)
1
= (20) cm2 = 10 cm2 Choice (A)
2

19. Let the side of the square be a units and that of the regular
hexagon be x units
4a = 6x  a = 3/2 x
Now, required ratio = a2 : 3 3 x 2
2
2
3  3 3 2
=  x   x =3:2 3 = 3 :2 Choice (D)
2  2

20. Side of the inscribed hexagon


= Radius of the circle = 10 cm
 Area of the inscribed hexagon = 3 3 (102)
2

= 3 3 (100) cm2 = 150 3 cm2


2
Also, Radius of the circle = Distance between the parallel
sides of the circumscribed hexagon = 2(10) = 20
 3 a = 10, (a is the side of the circum hexagon)
2
 a = 20/ 3
Triumphant Institute of Management Education Pvt. Ltd. (T.I.M.E.) HO: 95B, 2nd Floor, Siddamsetty Complex, Secunderabad – 500 003.
Tel : 040–40088400 Fax : 040–27847334 email : [email protected] website : www.time4education.com SM1002162/41
2 26.
3 3  20 

 Area of the circumscribed hexagon =  D E
2  3  C
 
3 3  400 
=   cm2 = 200 3 cm2 F
2  3 
 Required difference = (200 3 – 150 3 ) cm2
= 50 3 cm2 Choice (B)
A B
21. Horse can cover 3 sectors of sector angles 270°, 90° and 90°.
(1) Area of the field of sector angle 270°
Area of  ABE =
1
 BEAF
2
1
= (36)(25)  450
2
7 Area of the square ABCD = 2(450) = 900. Ans : (900)
7
27. AB = 5, BC = 6, CA = 7 A
567
s= 9
2
= 9 ( 4 )( 3 )( 2 ) = 6 6
270  22  2 2
=   14 = 462 m
360   7  Inradius, r =   6 6
s 9
(2) Combined area of the field of sector angles 90° B C
= 2 6
= 2  90 22 (7)2  = 77 m2. 3
 360 7 
Area of the circle =   4  6 =
8
Choice (B)
 Required area = 462 + 77 = 539 m2 9  3
Ans : (539)
28.
22. The figure is made up of trapeziums ABCD and ADEF
Area of the figure
1  1 
  (BC  AD)  BG   ( AD  EF )FH
2  2 
r
1  1  2
  (10  21)8   (21  8)12 cm
2  2 
= 298 cm2 Ans : (298)
60°
23. 4m Let the radius of the smaller circle be 'r'.
r
The radius of the bigger circle = = 2r
sin 60 3
4m Choice (B)

29. 20 cm
A B
Total area = (4)(50) + (4)(20) –4(4) m 2

= 264 m2 Choice (C)


h
24.
A

D 24 cm C

B O ½ (20 + 24)h = 220  h = 10 cm


Now, trapezium ABCD can be split into a rectangle and a
triangle as follows:
20
C
x x
In the given figure, OA = AB = BC = OC = OB 10 10
Let the sides be ‘a’ units each.
Then, Area of the rhombus = 2  (Area of the equilateral
2 2
triangle OAB)
x= 100  4  2 26 cm Choice (A)
3a 2 3a 2
=2 
4 2 30. P
B A
Now, 3 2 =8 3 a=4
a
2
 Area of the circle = a2 = 16 cm2. Choice (D)
25. Perimeter of a sector (p) = l + 2r D
C Q
 p = l + 2r = 64  l = 22 cm
 Area of the sector = ½ lr Let the length of AB be 4x and the breadth AD be 3x
= ½(22)(21) cm2 = 11(21) cm2 = 231 cm2 Ans : (231) Now 4x  3x = 768  x = 8
Triumphant Institute of Management Education Pvt. Ltd. (T.I.M.E.) HO: 95B, 2nd Floor, Siddamsetty Complex, Secunderabad – 500 003.
Tel : 040–40088400 Fax : 040–27847334 email : [email protected] website : www.time4education.com SM1002162/42
 AB = CD = 4  8 = 32 cm  r = 3.5 cm  h = 3  3.5 = 10.5 cm
AD = BC = 3  8 = 24 cm 22
Now, PB = ¼  32 = 8 cm  Volume =  (3.5)2  10.5 = 404.25 cu.cm
7
Let CQ = y cm Choice (D)
½ (8 + y)  24 = 288  8 + y = 24  y = 16 cm
 CQ = 16 cm Ans : (16) 38. Volume of the frustum
31. Volume of the gold in the pipe

3

= (102 )(10)  5 2 10 =
3
875
3
 
= (72  62)  40 = 11440 = 16342/7 cu.cm 5
Combined volume of the two cylinders
11440
Weight of the gold =  21 = 34320 gm = 34.32 kg =  (52)( 5) +  (102)( 5)= 625 5
7
875
Volume of bronze =   6  40
2 Total volume =  625 10
3
31680 5
=  4525 cu.cm 2750
7 7 = cubic cm Choice (D)
3
31680
 Weight of bronze =  28 = 126720 gm
7 39. A
= 126.72 kg
Total weight = 34.32 + 126.72 kg = 161.04 kg
Ans : (161.04) P Q

32. Volume of the water in the conical tank is equal to the volume U R
of the water in the frustum.
1 1 B C
 ()(7)2(h) = [62 + (6)(3) + 32]14. T S
3 3
(36  18  9)(14) In the given figure; PU = UT = PQ
h= = 18 m. Ans : (18) But UT = BU and PQ = PA
49
 PA = PU = BU
33. Let the radius and the slant height be r units and ℓ units  PU = 12/3 = 4 cm
respectively.
ℓ + r = 2 (Difference of 2r and ℓ)  Area of hexagon PQRSTU = 3 3  (4)2
2
If 2r = ℓ, ℓ + r = 0 which is not possible.
If 2r > ℓ, ℓ + r = 2(2r – ℓ) = 24 3 cm2 Choice (B)
ℓ = r which is not possible.
If 2r < ℓ, ℓ + r = 2(ℓ – 2r) 40. The area of the triangle formed by joining any three alternate
vertices of a regular hexagon will always be half of that of the
ℓ = 5r  r 2  h2  5r hexagon. As Q, S and U are alternate vertices of PQRSTU,
required ratio is 2. Choice (C)
Squaring both sides, h2 = 24r2  h = 2 6r Choice (A)
41.
P Q
34. Let the edges of the cubes be 3x, 4x and 5x cm
12 3 V
Side of the new cube = = 12 (Diagonal = 3 a)
3 T
W
 (3x)3 + (4x)3 + (5x)3 = 123
1728
 (27 + 64 + 125) x3 = 1728  x3 = =8x=2 S U R
236
 Edge of the smallest cube will be 3  2 = 6 cm As VT = VU, V lies on the diagonal PR.
Ans : (6) As RV = 2PV, RV = (2/3) PR
35. Let radius = r cm and height = h cm RW = (1/4) PR  VW =  2  1  PR  5 PR
r + h = 21 and 2r(r + h) = 924 3 4 12

Area of  RUT 3
 22  RW 3
r 21 = 924  12 
= Choice (D)
 2 Area of  VUT VW 5 5
 7  12

r= 924(7) = 7  h = 21  7 = 14
42. The given rectangle is shown in the figure below.
2(22)(21)
22 8 G 24
 Volume = (7)(7)(14) cm3= 2156 cm3 Choice (A) A B
7
10 6
36. Volume of the barrel = 22  1  1 (7 ) = 51/2 cu.cm
  
7  2  2  F
Number of words written using 51/2 cu.cm of ink is 2200. 18 18
 Number of words written with 200 ml (200 cu.cm)
20
2200
i.e. 200 cu.cm =  200 = 80,000 Ans : (80000)
11/ 2
E 8
37. h = 3r C
Total Surface Area = 2r(r + h) = 8r2 D
Cost of painting = 8r2  4.25 24 6
 8r2  4.25 = 1309
1309  7
 r2 = = 12.25 H
8  22  4.25 I

Triumphant Institute of Management Education Pvt. Ltd. (T.I.M.E.) HO: 95B, 2 Floor, Siddamsetty Complex, Secunderabad – 500 003.
nd

Tel : 040–40088400 Fax : 040–27847334 email : [email protected] website : www.time4education.com SM1002162/43


Statements , . The length of longest diagonal in the
It can be re–arranged to form a square. hexagon is twice the length of side, so side of the hexagon
 ADE slides along AE, AE slides to FFI.  AGF can be cut  3 
out and placed on  ECH. is 10 cm area of the hexagon is 6  a2  .
 4 
The perimeter of the square = 24  4 = 96 Choice (A)  
 3
43. Let us say there are x tiles along the length of the floor and Area of the hexagon = 6   (10 2 ) cm2.
y tiles along the breadth of the floor, excluding the tiles along  4 
the corners.  
Number of blue tiles = 2x + 2y + 4 Sufficient. Choice (C)
Number of green tiles = xy
xy = 2(2x + 2y + 4) 48. Statement  alone is not sufficient, as it gives no numerical
xy = 4x + 4y + 8 data.
xy  4x  4y + 16 = 24 Statement  alone is not sufficient, as we don’t know the
(x  4) (y  4) = 24 information about the other side or any of the angles of the
xy triangle.
x4y4 From  and , we have
Possibilities for (x  4, y  4) are (24, 1), (12, 2), (8, 3) and (6, 4). BAC = 45°, AC = 16 cm, BC = 12 cm.
 x is 28 or 16 or 12 or 10. Choice (A) B
D
44.

A B C 45° A
7m
16 cm
10.5m 7m Let D be the foot of the perpendicular from C to AB.
D AB = AD + DB
C

= CD + BC 2  CD 2  8 2  122  8 2  2

Let us say the cow is tied at the corner denoted by A shown


= 8 2 4
in the figure above. The area in which the cow can graze is
270 2 (90)
 10.52   3.52
1
= Area of ABC = (AB)(CD)
360 360 2
 3.52  3  9  1   11  2.5 3.5  29  CD = AC Sin 45° = 16  8 2 cm ≃ 11.3 cm. Sufficient.
=    7  2  2
2  2       Choice (C)
= 279.125 Ans : (279.125) 49. Let the length, breadth, and the height of the cuboid be
45. As AC and CE are the diameters, the volume of the solid ℓ cm, b cm and h cm respectively.
generated is the total volume of 2 spheres. We need to find 2(ℓb+ ℓh+bh).
Volume of the solid generated
3 Statement , ℓ 2+b2+h2 = 262
4 4 5 From this we can’t find the total surface area as we have one
= ( 5 3 ) +   
3 3 2 equation with three unknowns. Not sufficient.
4 375  Statement , ℓ + b + h = 38
=  (53) 1 1  = Choice (D)
3  8 2 From this we can’t find the total surface area as we have one
equation with three unknowns. Not sufficient.
Solutions for questions 46 to 50: Statements , . (ℓ + b + h)2 = 382

46. 2  r = 44  r = 7  ℓ 2 +b2 + h2 + 2(ℓb + ℓh + bh) = 382  2 (ℓb+ ℓh+bh)


Total surface area = 2  r (r + h) = 382 – 262. Sufficient. Choice (C)
Statement .
50. Statement . BAC = 60°.
r + h = 21
Hence the height of the cone
Substituting r and r + h values in 2  r (r + h) we can find the A
3 .5
total surface areas. Sufficient. = cm.
tan 60
Statement .
total surface area = curved surface area + 2r2 As we know the height and the radius
As r = 7 of the base, we can find the volume of
Sufficient. Choice (B) the cone. Sufficient.

47. Statement . We do not know the number of sides of the Statement . The ratio of the TSA
polygon, so we can’t find the area of the polygon. Not (in cm2) to the volume (in cm3) is 1.5 : 1 B 3.5 C
sufficient.
Statement . Each exterior angle is 60o, so the number of Let ℓ cm and h cm be the slant height and the height of the
360 cone respectively.
sides is , viz 6.
60  ℓ 2 – h2 = (3.5)2 ……. (1)
But we do not know the side of the polygon. Not sufficient. r (r  ) 3
Given, =
1 2 2
r h
3

Triumphant Institute of Management Education Pvt. Ltd. (T.I.M.E.) HO: 95B, 2nd Floor, Siddamsetty Complex, Secunderabad – 500 003.
Tel : 040–40088400 Fax : 040–27847334 email : [email protected] website : www.time4education.com SM1002162/44
2(r + ℓ) = rh  7 + 2ℓ = 7/2 h ……. (2) Sufficient. Choice (B)

Solving (1) and (2), we get ℓ and h.

Triumphant Institute of Management Education Pvt. Ltd. (T.I.M.E.) HO: 95B, 2nd Floor, Siddamsetty Complex, Secunderabad – 500 003.
Tel : 040–40088400 Fax : 040–27847334 email : [email protected] website : www.time4education.com SM1002162/45

You might also like